NEURO1 - Dr. Robbins Cases Copy

¡Supera tus tareas y exámenes ahora con Quizwiz!

Hyposomia is the earliest manifestation of : 1. Aging 2. Parkinson's disease 3. Sinusitis 4. Déjà vu 5. Foster Kennedy syndrome

(2) Recent research has found the hyposomia is the earliest finding in Parkinson's disease and Alzheimer's disease. Tests are underway to see if this can be used in early detection and thus early treatment of these diseases.

A 32-year-old woman is seen in the neurology outpatient clinic with symptoms of diplopia and ptosis that fluctuate during the course of the day. Symptoms increase with reading. Examination shows the ptosis increases with sustained upward gaze. Routine laboratory studies were normal. Which of the following statements concerning the above case is true? 1. It is an autoimmune disorder caused by antibodies that are directed against presynaptic nicotinic acetylcholine receptors. 2. It is an autoimmune disorder caused by antibodies directed against postsynaptic muscarinic acetylcholine receptors. 3. It is an autoimmune disorder caused by antibodies directed against presynaptic voltage-gated calcium channels. 4. It is an autoimmune disorder caused by antibodies directed against postsynaptic nicotinic acetylcholine receptors. 5. It is an autoimmune disorder caused by antibodies directed against the synaptic enzyme acetylcholinesterase.

(4) It is an autoimmune disorder caused by antibodies directed against postsynaptic nicotinic acetylcholine receptors. The primary antigenic target in autoimmune myasthenia gravis is the post synaptic nicotinic acetylcholine receptor. Presynaptic voltage-gated calcium channels are the target of the Lambert-Eaton myasthenic syndrome.

A 62yo woman presents for eavulation of severe pain and presthesias in her feet. Sensory examination shows impairments to pinprick and temperature below the ankles. Motor examination and reflexs are preserved. A small fiber neuropathy is considered. Which of the following is incorrect regarding this condition? 1. Diabetes is a common cause fo this condition 2. Quantitative sudomotor axon reflex test can be used for diagnosis 3. Skin biopsy and intra-epidermal nerve fiber density can be used for diagnosis 4. Thermoregulatory sweat gland test can be used for diagnosis 5. EMG/NCS are usually abnormal

(5) EMG/NCS are usually abnormal = these tests are usually NORMAL in pts with pure small fiber neuropathy. The samll fibers include myelinated A and unmyelinated C fibers which are involved in autonomic, tempearture and pain transmission. Patients with small fiber neuropathy present with painful burning sensations na dysethesias distall (MC in feet). Some pts may have autonmic manifestations that compromise sweating, vasomotor control, GI and GU fx. In pure smal lfiber neuropathy, besides sensory findings the neuro exam is normal (reflex and motor test), distinguishing it from large-fiber neuropathy.

A 45 year old female presents with ptosis and a mild intermittent diplopia. She noticed this 3 years ago however it is becoming more persistent. She denies any other weakness with swallowing, breathing, or in her extremities. The ptosis is not present when first waking in the morning. But is now causing her problems at work as a secretary. Exam demonstrated only increasing ptosis when looking up for 1 minute, the pupils are normal and the remaining neurological exam is normal. Based on the above clinical presentation. The following characteristics of this disorder is not true: 1. You would expect low acetylcholine receptor antibodies levels 2. No other generalized symptoms for 2 years in order to be diagnosed 3. Treat with AChE inhibitors initially 4. Immunotherapy will eventually be needed 5. Symptoms should resolve eventually

(5) Symptoms do not resolve eventually and treatment will be required the rest of her life. Acetylcholine receptor antibodies are typically low or normal on initial exam. So you would start by confirming your diagnosis by EMG of the facial muscles (orbicularis oculi) and or the Tensilon test. You would then start treatment with pyridostigmine 60 mg TID to QID, as tolerated. You would recheck her in 2 weeks to see if the symptoms are controlled

A movement disorder characterized by repetitive involuntary movements of the mouth, lips and tongue and occasionally accompanied by dystonic posturing or choreoathetotic movements of the limbs and trunk and often occurs as a complication of phenothiazine therapy is: A. Focal dystonia B. Huntington's chorea C. Tardive dyskinesias D. Sydenham's chorea E. Myoclonus

(C) Tardive dyskinesias.

A 38-year-old previously healthy man consulted a neurologist because for the previous 6 months he had experienced episodes of clonic movements starting in the left thumb, spreading to the left arm and face. Each event lasted 1 to 2 minutes. The patient was alert and responsive throughout each episode and had a good recollection of them. On examination, he had mild clumsiness of his left hand and hyperactive muscle stretch reflexes in his left arm. What is the diagnosis? 1. Simple partial seizure 2. Partial complex seizure 3. Absence seizure 4. Generalized major motor seizure 5. Status Epilepticus 6. Myoclonic seizures

1 - Simple partial seizure

A 21yo right-handed woman experienced a sensation of numbness and tingling over her abdomen and in her legs. The next day her legs began to feel stiff and tight, and she experienced incomplete voiding and difficulty in initiating her urinary stream. As the day progressed, the numbness and tingling became more pronounced in her mid-‐abdomen and below, and she noted difficulty in walking. She went to bed early that evening, and when she awoke the next morning, she was unable to stand. 1. Where is the lesion? 2. Is the lesion focal, multifocal, or diffuse? 3. Is this a mass lesion or a non-mass lesion? 4. What is the temporal profile? 5. What diagnostic category is most likely

1) Even though no information about the physical examination is available, the likely lesion localization can be inferred from this patient's history alone. The abnormal sensory and motor function below a level in the mid-abdomen indicates a focal lesion in the spinal cord, at the thoracic level or above (RULE #3). 2) Focal 3) Since the symptoms are progressive, this is a mass lesion. 4) The time course is subacute. 5) A focal, subacute lesion is typically inflammatory, specifically, an abscess. Comment: An MRI scan of the cervical and thoracic spine failed to demonstrate an abscess, but a lumbar puncture revealed a pleocytosis in the spinal fluid. All cultures were negative. The patient was thought to have transverse myelitis, and inflammatory (possibly autoimmune) disorder that behaves like a mass lesion within the spinal cord. Her symptoms gradually resolved without treatment. This is another example of how the rules can fail. In this case, they pointed to the correct diagnostic category but the wrong specific diagnosis. As it happens, it is often impossible to distinguish between transverse myelitis and a spinal cord abscess on clinical grounds alone, so it is appropriate to direct the evaluation at the possibility of an abscess, which would require urgent treatment.

A 72yo right-handed man noted the abrupt feeling of heaviness in his left arm while watching television. His left leg gave out when he tried to stand, and he fell to the floor. He called for help, and when his wife came into the room, she noted that the left corner of his face was sagging. He could still speak. He had no symptoms other than the weakness of his entire left side. Over the next several hours he improved slightly. 1. Where is the lesion? 2. Is the lesion focal, multifocal, or diffuse? 3. Is this a mass lesion or a non-mass lesion? 4. What is the temporal profile? 5. What diagnostic category is most likely?

1) Facial weakness ipsilateral to body weakness suggests a focal lesion high in the pons or above. (See RULE #6 from the previous lecture) 2) Focal 3) There has been no progression, so this is not a mass lesion. 4) The symptoms developed acutely 5) A focal lesion that developed acutely and is improving is usually a vascular lesion.

A patient reports a pain that is similar to the one described in the preceding case (CASE 7). The pain involves the left side of her chest. No rash is present and for many months, the pain has been getting worse. It remains localized to a narrow and circumscribed area of her chest, making her think that it might be "heart trouble." In addition, she complains of difficulty walking and says that her le leg seems to be weak and stiff. 1. Where is the lesion? 2. Is the lesion focal, multifocal, or diffuse? 3. Is this a mass lesion or a non-mass lesion? 4. What is the temporal profile? 5. What diagnostic category is most likely

1) In addition to chest wall symptoms like those described in the patient in Case #10 this patient has left leg weakness, which could not be explained by a lesion in the thoracic nerve root. Instead, the most plausible localization is within the thoracic spinal cord on the left, in the one or two segments where the spinothalamic pathway has not yet crossed to the right side of the cord. 2) This is a focal lesion 3) Since it is progressive, it is a mass lesion. 4) The time course is chronic. 5) A chronic focal lesion is a neoplasm. A meningioma was removed and the patient is doing well after 3 months of physical therapy.

A 30-‐year-‐old man with Hodgkin's disease began to experience severe pain beginning in his back and encircling the left side of his chest in a band 3 cm wide just below his breast. The pain was very intense at first but subsided somewhat, coincident with a rash that appears in precisely the same distribution. He is still having pain in that area two weeks later, and notes diminished touch sensation in the region of pain. 1. Where is the lesion? 2. Is the lesion focal, multifocal, or diffuse? 3. Is this a mass lesion or a non-mass lesion? 4. What is the temporal profile? 5. What diagnostic category is most likely

1) The patient has symptoms confined to the distribution of a single nerve root. This makes a lesion in the nerve root itself most likely, but theoretically the lesion could be anywhere from the level of the nerve root on up the sensory pathway to the cortex. However, additional symptoms would be expected with any of these higher localizations. 2) In any case, the lesion is focal 3) The lesion is not progressive, so it is not a mass lesion. 4) The time course is subacute 5) The process is probably inflammatory, and probably due to shingles. Comment: A rash in a dermatomal distribution is very suggestive of herpes zoster reactivation, which can produce sensory symptoms (especially pain) in the same distribution. This occurs particularly often in immunocompromised individuals, including those receiving chemotherapy for cancer. The patient received famciclovir and pain treatment, and the symptoms gradually resolved.

A 57yo woman has come to the emergency room because in the middle of a business meeting earlier today she suddenly became dizzy and experienced nausea and vomiting. On examination, she has dysarthria, dysphagia (with weakness of the left palate), loss of pinprick sensation over the left side of her face and right side of her body, and ataxia of her arm and leg. Seven hours have passed and there has been no improvement. 1. Where is the lesion? 2. Is the lesion focal, multifocal, or diffuse? 3. Is this a mass lesion or a non-mass lesion? 4. What is the temporal profile? 5. What diagnostic category is most likely

1) The reduced pinprick sensation over the left face and right body imply a focal lesion on the left, between the pons and the C2 level of the spinal cord. (See RULE #5). The weakness of the left palate confines the lesion to the medulla, because only lower motor neuron lesions produce unilateral palatal weakness. Ataxia of the left arm and leg is also consistent with a lesion in this location, because a lesion in the medulla can disrupt cerebellar connections. 2) Focal confined to the left medulla 3) The symptoms began acutely 4) The symptoms have not progressed, so this is not a mass lesion. 5) A focal, acute lesion with a static time course implies either a vascular (ischemic) or traumatic etiology, and there is no history of trauma in this case. Comment: This patient had a normal head CT scan, but MRI reveals a small infarction in the left lateral medulla. In the emergency room, she was found to have atrial fibrillation, which had never been noted before. The stroke was presumed to be embolic, so anticoagulant therapy was begun. Further evaluation revealed hyperthyroidism. She was converted to normal sinus rhythm, her hyperthyroidism was treated, and after 6 months the anticoagulants was stopped. By that time, her symptoms had completely resolved except for minimal ataxia of the left are more than likely this is due to vascular disease of the PICA. If a Horner's syndrome was found then you have a Wallenberg's Syndrome due to vascular insufficiency involving the lateral medullary artery. It's the only syndrome in the brainstem that I want you to know.

A 47yo man developed ringing in his right ear several years ago, and it has grown worse over time. His hearing in that ear also has gradually deteriorated. Over the same time period, he has developed weakness and loss of feeling in the right side of his face, and he now notes stiffness, weakness, and numbness of his left arm and leg 1. Where is the lesion? 2. Is the lesion focal, multifocal, or diffuse? 3. Is this a mass lesion or a non-mass lesion? 4. What is the temporal profile? 5. What diagnostic category is most likely

1) The right facial numbness and left body numbness indicates a lesion on the right between the pons and C2 level of the spinal cord (RULE #5). The left arm and leg weakness and stiffness further restrict the possible localization to the region between the pons and the low medulla. Finally, the tinnitus and hearing loss in the right ear localize the lesion further, to the right pons or pontomedullary junction. 2) Such a precise unilateral localization implies a focal lesion. 3) It is progressive so it is a mass lesion. 4) The progression has taken place over several years, making it chronic 5) A focal, chronic lesion is a neoplasm Comment: In this case, the neoplasm was an acoustic schwannoma (a benign tumor). It was resected, and the patient continues to do well 15 years later.

A complete paralysis at C5-C6 causes which of the following: 1. Complete paralysis of legs, arms can still abduct and flex 2. Diaphragmatic paralysis (no respirations) 3. Complete paralysis of legs, and only the hands 4. Complete paralysis of legs, and dermatome level 5. Complete paralysis of arms and legs

1, Complete paralysis of legs, arms can still abduct and flex due to the intact C5 nerve root which allows the deltoid and biceps to function, hence, the arm can abduct and flex. More distal function at C6 and below is lost.

A 27-year-old woman in the second trimester of hr. first pregnancy was referred to the epilepsy clinic from the high-risk obstetrics clinic. As a college student at age 19 years she had her first generalized tonic-clonic seizure. She had three more seizures during her early twenties, each beginning with an unpleasant sensation in her abdomen, followed on one occasion by staring and picking at her blouse buttons before progressing into a generalized tonic-clonic seizure. She was the product of a normal pregnancy and delivery with normal developmental milestones. At age 15 months she had experienced two prolonged seizures with high fever, both lasting approximately 15 minutes and associated with transient paralysis of her right arm. She had continued to develop normally and had been successful both athletically and scholastically. Since age 24 years she had taken phenytoin, 200 mg twice a day. 1. What type of seizures does the patient have? 2. What diagnostic tests does this patient need? 3. What are the drugs of choice for this patient? 4. How should this patient be managed? ----->Through the rest of her pregnancy? ----->And thereafter?

1. Based on her history, this young woman has seizures that are simple partial at onset, progressing to complex partial, and then progress to secondarily generalized tonic-clonic seizures. Her seizures are probably a result of the prolonged complicated febrile convulsions when she was age 15 months. 2. Unless they were obtained previously, she should have EEG studies looking for focal slowing and interictal epileptiform activity and a brain MRI scan looking for a structural cause of her seizures. In this instance the brain MRI should be delayed until after delivery. A search for a toxic-metabolic cause of seizures is unnecessary given the long history. 3. Teratogenic effects of AEDs occur during the first trimester (within the first 25 days of conception). Because this patient was first seen during her second trimester, it would be prudent to continue drug therapy at the current dose. 4. ----> The patient should be seen frequently during the remainder of her pregnancy. Free levels of phenytoin should be checked regularly. The dose should be adjusted to maintain a stable free level, and in the last month of pregnancy the patient should take oral vitamin K. She has been seizure-free for 4 years, so she might be a candidate for AED withdrawal, but this should be deferred until she is in a relatively stable physiologic state. She is likely to be sleep deprived for at least several months after delivery, and sleep deprivation is a common precipitant of seizures in predisposed patients 4. ----> After delivery, she should return to her pre-pregnancy phenytoin dose by the first month postpartum. An MRI scan should be obtained as mentioned above, and regular follow-up under the supervision of a neurologist should be arranged. About 6 months after delivery, when her metabolism has returned to baseline and her child's sleep schedule has been established, her physician can discuss the advantages and disadvantages of AED withdrawal with her. She will have to weigh the inconvenience of taking daily medications, and the potential side effects (including teratogenic risk if she becomes pregnant again in the future), against the risk of having a seizure while caring for a young child and the inconvenience of refraining from driving while tapering her medication and for some months thereafter. She must ultimately decide which of these considerations are most important to her; she should be reassured that the physician will support her decision either way. If she chooses to remain on an AED, she should take folic acid, 1 mg daily, to reduce the risk of major teratogenic side effects in the event of future pregnancy.

A 31-year-old woman with no family history of neurologic disease and no history of trauma or drug abuse had the spontaneous onset of a horizontal ("no-no") head tremor. Initially, it was mild and asymptomatic, but as it increased in severity, the patient noted that turning her head to the right seemed to increase the tremor, whereas turning her head to the left decreased the tremor. Her neurologic examination results were otherwise normal, as well as her MRI. Beta-blockers, primidone, and alcohol had little effect on the tremor. Eighteen months later, the patient returned, reporting that driving had become difficult because of a tendency for her head to turn involuntarily to the left, and her attempts to hold her head in the neutral position markedly increased the tremor and caused discomfort in her neck. Stress also caused similar problems. Over time, the involuntary movements were present almost continuously. Mild finger pressure on her left chin dampened the tremor and the involuntary movements of her head. What is the diagnosis? 1. Dystonic tremor 2. Psychogenic tremor 3. Chorea 4. Familial tremor 5. Essential tremor

1. Dystonic tremor. If the tremor was absent and the movement was that of a sustained contraction then a focal dystonia (spasmodic torticollis) would have been the diagnosis: Neurologic examination demonstrated hypertrophy of the right sternocleidomastoid muscle. A trial of anticholinergic medication increased the tremor and precipitated a psychotic reaction. Eventually, the patient was treated with botulinum toxin in the right sternocleidomastoid muscle and some of the left paracervical muscles. This treatment controlled the involuntary movements and dampened the involuntary tremor.EtOH attenuates essential tremor. Not familial b/c not essential.

This 40-year-old male is seen by you in your office regarding tremor in his hands. He states he has always been nervous and shakes a lot when stressed. He states the tremor is causing him problems at work. He is a draftsman with an architecture firm for the past 20-years. He notices increase tremor with 1-2 cups of caffeine but the tremor seems to dissipate some when he is off work and has a couple of drinks. Your examination demonstrates tremors in his hands when he is using a pen to write (you have him write his name), when his hands are out stretched you observe a fine tremor of the fingers. The tremor abates with him at rest. The rest of the examination is normal. Your diagnosis and treatment is. 1. Essential tremor - beta-blocker 2. Writing Dystonia - clonazepam 3. Parkinson's tremor - acetyl cholinergic drugs 4. Cerebellar tremor - stop alcohol 5. Physiologic tremor - stop caffeine

1. Essential tremor: the treatment is first a beta-blocker (Inderal or Lopressor). if not able to take then Mysoline (Primidone). Primidone breaks down into phenobarbital so it can be sedating, you have to start with low dose and titrate slowly until control is achieved.

A patient presents with a generalized chorea-form disorder that began at the age of 45. He denies any neurologic or psychiatric problems prior to the onset of his current problem and never received neuroleptic medications. He denies a family history of any similar movement disorder, but his mother was institutionalized at the age of 55 for psychiatric reasons. What is this patient's possible diagnosis? 1. Huntington's disease 2. Parkinson's disease 3. Essential tremor 4. Idiopathic torsion dystonia 5. Sydenham's chorea

1. Huntington's disease is an autosomal dominant disorder with onset typically in middle age. Although psychiatric symptoms are insufficient for making a diagnosis of Huntington's, a family history of a parent with psychiatric disease in a patient with choreiform disorder may be very suggestive. The diagnosis can be made through genetic testing.

A patient has weakness of his hip flexion, knee flexion, and ankle dorsiflexion in the left lower extremity. Based on this limited amount of information. Answer BOTH of the questions before you scroll below for the answer. 1. Is this distribution of weakness consistent with a single root lesion? 2. Is this pattern of weakness suggestive of any other lesion localization?

1. NO; The L2-L3 roots correspond to hip flexion, the L5-S1 roots to knee flexion, and the L4-L5 roots to ankle dorsiflexion. Thus, no single root lesion could produce this pattern of weakness. 2.This pattern is typical of an upper motor neuron lesion, which usually produces more weakness in the flexors of the lower extremity than in the extensors. You don't have enough information to determine the level. With this limited information you cannot accurately pin point the level, what other information do you need? Good exercise for your brain? ANSWER: You would need to know what the reflexes were and what sensory findings are present. If they are decreased then you would suspect multiple nerve root or plexus pathology. If increased then you would consider the spinal cord as a possibility.

A 55-year-old woman had a generalized tonic-clonic seizure while grocery shopping one afternoon. She had been experiencing moderately severe headaches early every morning for approximately 1 week but had been well previously. When the emergency medical service arrived, the patient was having a second generalized tonic-clonic seizure. A witness reported generalized stiffening of all four extremities followed by clonic movements associated with cyanosis, frothing at the mouth, and urinary incontinence. The patient had two more seizures without recovering consciousness before reaching a local emergency room. The emergency room physicians diagnosed convulsive status epilepticus. 1. Do you agree with the emergency room doctors' diagnosis? 2. How should this patient be managed in the emergency room? 3. How should this patient be investigated after her seizures are controlled?

1. The emergency room physicians' diagnosis of convulsive status epilepticus is correct. This patient had 4 tonic-clonic seizures with failure to return to normal consciousness in between, which certainly satisfies the definition of status epilepticus. 2. Emergency management of the patient should proceed as previously discusses in slide 35. 3. Even if seizures cease rapidly, all patients who have just been treated for status epilepticus should be admitted to an intensive care unit for at least 24 hours of close observation. An epileptic disorder presenting de novo as status epilepticus requires immediate investigation. In this patient, the likelihood of finding a significant underlying problem is high, especially given the history of headaches for the past week. An MRI scan in this patient revealed multiple areas of signal abnormality that enhanced with contrast. The patient proved to have multiple small cerebral metastases from a primary tumor that was never identified.

A 7-year-old boy was noted by his teacher to be intermittently inattentive in the classroom. He would stare with a blank expression on his face for several seconds at a time. He would not respond to his name being called and sometimes had rapid fluttering movements of his eyelids. Once the staring ceased, he would immediately become his usual self. His pediatrician was able to provoke one of the spells by having the child hyperventilate in the office. 1. What is the most likely diagnosis in this patient? 2. What other diagnoses need to be considered? 3. What is the drug of choice in this syndrome?

1. The history is typical of absence seizures. The provocation of the attacks by hyperventilation is also characteristic. The EEG findings were bursts of generalized 3-Hz spike-and-wave activity occurring spontaneously and provoked by hyperventilation would confirm the diagnosis. 2. Other entities to be considered in the differential diagnosis would be complex partial seizures (which can also be associated with motionless staring with no other symptoms, but are less readily provoked by hyperventilation), and attention deficit disorder, or some other form of behavioral disturbance. 3. In view of the normal neurologic examination and the generalized nature of the child's EEG discharges, no imaging studies are indicated. This child was treated with ethosuximide, introduced slowly and built up to a dose of 750 mg (25 mg/kg for this 30 kg boy) per day. This drug is well tolerated and eliminates both the seizure activity and the EEG discharges in almost all cases. The outlook for normal neurologic and intellectual development in a patient such as this is excellent.

In the dark, a patient's left pupil is 3 mm larger than the left. In bright light, the pupil is only 1 mm larger than the left 1. Which pupil is abnormal, the right or the left? 2. Which pathway is abnormal, the sympathetic or the parasympathetic? 3. Is this patient likely to have ptosis? If so, on which side?

1. The magnitude of this patient's pupillary asymmetry is greatest in the dark, so the sympathetic system is not functioning normally in the right eye. The smaller pupil is therefore the abnormal one. 2. Thus this patient has a sympathetic lesion on the right. 3. A right sided ptosis would be expected. ...probably a Horner's syndrome (you have 2 of the 3 components for the diagnosis, meiosis and ptosis....you only need anhydrosis to make it complete). Comment: With sympathetic lesions, the pupillary asymmetry is greatest in the dark, whereas with parasympathetic lesions, the asymmetry is greatest in bright light.

A 65 year-old patient presents with a sudden onset of speech difficulties and a weakness in his left hand. Your examination finds a receptive Dysprosody and the patient seems to deny that he has any problem even though he is unable to use his right upper extremity. You attempt to show him that he is unable to use his arm by picking it up and asking him to move his fingers, which he is unable to do. He still claims that nothing is wrong. This patients demonstrates: 1.Anosognosia 2.Apraxia 3.Tactile agnosia 4.Prosopagnosia 5.Visual extinction

Anosognosia - due to a parietal lobe damage in the non-dominant hemisphere

A 27-year-old woman presented with a 7-year history of 6 episodes of feeling of confusion followed by repetitive lip smacking, chewing motion, and semi purposeful moments of her hands. During these events, she did not respond to questions appropriately. Each episode lasted 5 minutes. She was amnestic for most of the time during each seizure and confused for 1 to 2 hours after each seizure. Between episodes, she felt well. Her medical history included a few febrile convulsions at the age of 2 years. The neurologic examination results were normal. What is the diagnosis? 1. Simple partial seizure 2. Complex partial seizure 3. Absence seizure 4. Generalized major motor seizure 5. Status Epilepticus 6. Myoclonic seizures

2- Complex partial seizure not partial b/c amnesia not absence b/c longer than 30sec

Axonal loss is characterized by the following: 1. Increase of the SNAP and CMAP amplitudes. 2. Normal NCV. 3. The earliest sign of denervation is increased recruitment, followed by appearance of fibrillation potential and positive sharp waves in 1 - 4 days. 4. Reinervation with small MUPs with high amplitudes within several months.

2. Normal NCV.

Demyelination is characterized by: 1. Shortened distal motor latency 2. Slowing of the NCV to less than 80 percent of normal. 3. Normal F wave 4. Conduction block and decreased temporal dispersion. 5. Normal H reflex

2. Slowing of the NCV to less than 80 percent of normal.--- Criteria for determining if demyelination is present: (1) Prolonged distal motor latency. (2) Slowing of the NCV to less than 80 percent of normal is consistent with demyelination. (3) Conduction block* and increased temporal dispersion (a result of a failure to activate the next node of Ranvier). (4) H and F waves would be prolonged *Conduction block is defined as a 50% or greater decrease in amplitude of the motor potential upon proximal simulation and is a classical sign of focal demyelination and axonal loss.

A 10-year-old boy was brought to his pediatrician because of poor attention in class. The teacher noted that the child had brief (less than 15 seconds) episodes of impaired alertness and responsiveness. Some eye flutter and occasional facial twitching were noted during the episodes. The child seemed normal on termination of the episodes and was unaware of any problem. The family history was positive for seizures in a cousin. The neurologic examination results were normal. EEG demonstrated the classic 3-Hz spike-and-wave pattern. What is the diagnosis? 1. Simple partial seizure 2. Partial complex seizure 3. Absence seizure 4. Generalized major motor seizure 5. Status Epilepticus 6. Myoclonic seizures

3- Absence seizure what if normal eeg? still dx and tx

A 72-year-old man reported that in the past 6 months he had experienced difficulty playing golf. He noted that when he stepped to the tee, he had increasing problems maintaining his balance because his legs became very tremulous. Although he could walk all 18 holes without difficulty, he was unable to maintain his balance when he tried to stand still to make his shots. To compensate, he assumed an ever-widening stance, but this became increasingly less helpful. Similarly, he had routinely begun to sit down when he urinated. In contrast, he found that he could still walk well, but when he stopped, he had to sit down because he was unable to maintain his balance while standing. Neurologic examination demonstrated an alert pleasant man with normal facial expression. He arose from his chair without difficulty, walking with a normal gait, including a good arm swing. However, when he stopped and tried to stand still, he stood with an abnormally wide base, soon with development of a tremor of both of his legs. It was necessary for him to hold on to someone to keep from falling. He had no tremor while seated at rest, there was no cog wheeling or rigidity, and his strength was normal in all four extremities, with normal muscle stretch reflexes and sensation, particularly position sense. MRI and EMG were normal. What is this patient suffering from? 1. Essential tremor 2. Parkinson's disease 3. Orthostatic tremor 4. Senile tremor 5. Cerebellar tremor (Holmes tremor)

3. Orthostatic tremor is often a misdiagnosed problem of late middle age that is characterized by a 16-25 Hz tremor affecting the lower extremities, typically with weight bearing. Isometric limb muscle contraction, the critical generation factor, induces the tremor. Often if the patient cannot sit down or resume walking, they become distressed and sometimes fall. Patients stand with a wide base but can walk normally as the tremor abates in the non-weight bearing extremity.

The wife of a 46-year-old man reported an event in which he suddenly cried out, extended his arms and legs, and then displayed rhythmic generalized jerking of all 4 extremities. He was unresponsive and unconscious, his face turned blue, and he later became limp. He bit his tongue and was incontinent of urine. Gradually, he woke up, reporting of muscle soreness. He was taken to his physician's office. Neurologic examination results were normal later that day. The patient had no recollection of the event. What is the diagnosis? 1. Simple partial seizure 2. Complex partial seizure 3. Absence seizure 4. Generalized major motor seizure 5. Status Epilepticus 6. Myoclonic seizures

4 - Generalized major motor seizure

The more sensitive test for myasthenia gravis is? 1. Repetitive nerve stimulation 2. F wave 3. Acetylcholine receptor antibody 4. Single fiber EMG 5. H reflex

4, Single fiber EMG. Generally the pa8ent has repetitive nerve stimulation preformed first and if inconclusive then Single fiber studies are carried out.

A 32-year-old man with a history of posttraumatic recurrent partial motor seizures, occasionally leading to generalized tonic-clonic seizures and drug noncompliance, was brought to the ED with recurrent seizures. His wife stated that the day before admission, he had experienced a few episodes of jerking of his hand. During the early morning hours, the patient's wife was awakened to find her husband having a generalized grand mal convulsion. Although to her he seemed to have been "coming out" of this event, he then proceeded to have another tonic-clonic seizure. Thereafter, he remained deeply stuporous. In the ED, his phenytoin level was 1.4µg/mL. He was treated with IV phenytoin. What is the diagnosis? 1. Simple partial seizure 2. Partial complex seizure 3. Absence seizure 4. Generalized major motor seizure 5. Status Epilepticus 6. Myoclonic seizures

5 - Status Epilepticus

A 17-year-old girl reported a history of unprovoked massive muscle jerks involving either arm for approximately 1 year. These tended to occur in the morning. This patient also had 2 recent unexplained falls. She did not report lapse of consciousness during her falls. Her neurologic examination results were normal. What is the diagnosis? 1. Simple partial seizure 2. Partial complex seizure 3. Absence seizure 4. Generalized major motor seizure 5. Status Epilepticus 6. Myoclonic seizures

6 - Myoclonic seizures

A 69-year-old man who has hypertension, hyperlipidemia, diabetes, coronary disease, and peripheral vascular disease undergoes an endovascular intervention for a thoracoabdominal aneurysm. After the procedure he has new neurologic findings attributed to an anterior spinal artery infarct of the spinal cord. What do you expect to find on examination? A. Paraplegia with bilateral loss of sensation to pain and temperature below the lesion, and preserved sensation to vibration and proprioception. B. Paraplegia with bilateral loss of sensation to vibration and proprioception below the lesion, and preserved sensation to pain and temperature. C. Loss of sensation to vibration and proprioception bilaterally, and preserved sensation to pain and temperature, with no weakness. D. Loss of sensation to pain and temperature bilaterally, and preserved sensation to vibration and proprioception , with no weakness. E. Weakness on one side with loss of sensation to vibration and proprioception ipsilaterally, and loss of sensation to pain and temperature on the contralateral side.

A, Anterior Cord Syndrome spares the dorsal columns (vibration and proprioception) and affects the anterior corticospinal (paralysis) and anterior spinothalamic tracts (loss of pain and temperature).

On the Frankel scale a level B would indicate which finding: A. Incomplete: some sensory preservation below the zone of injury B. Incomplete: motor and sensory sparing and the patient is functional (stands and walks) C. Complete functional recovery: the reflexes may be abnormal D. Complete : motor and sensory loss below the lesion E. Incomplete: motor and sensory sparing, but the patient is non functional

A, Incomplete injury with some sensory preservation below the zone of injury. (I WILL NOT ASK YOU THIS TYPE OF QUESTION ON THE EXAMINATION). This will recur on your Advanced Trauma Life Support course (ATLS), not required for your 3 year rotation. Some students take this course as an elective at some sites in preparation of ER rotations (again ATLS is not required to graduate from ATSU).

A 32 year-old woman presents to the ER complaining of blurred vision and pain in the right eye. Your evaluation shows decreased visual acuity in the right eye and funduscopic examination of both eyes is normal. There is a relative afferent pupillary defect on the right, and testing of the right visual field shows a small central scotoma. The most likely localization of the lesion is the: A. Choose from below. 1. Optic chiasm 2. Optic nerve 3. Optic tract 4. Occipital cortex 5. Optic radiations B. What do you call this finding?

A. Optic nerve. B. Retrobulbar optic neuritis = decreased visual acuity or blindness, afferent pupil, and a normal funduscopic Examination

A 22yo male was well until two days ago when he developed fever, severe HA, N/V. He became progressively more obtunded over the next day. He had two generalized seizures in the morning and was brought to the ER, where he was found to have a fever and stiff neck. He was stuporous and had generalized hyperreflexia and bilateral (+) Babinski's signs. What level? Focal or Diffuse? Mass or Non-mass? What is temporal profile? What general cause?

AMS (brainstem, thalamus, both cerebral hemispheres due to seizure); Diffuse or at multiple levels (due to symmetric hyperreflexia and Babinski); Non-mass; Subacute (2d); Meningitis or encephalitis.

QUESTION (Picture on next slide) = Previously well 30yo homosexual man presented with a 5d history of progressive bilateral ascending lower limb weakness, preceded by a flu-like illness lasting for 2wks. Examination revealed normal tone but slightly reduced power at 4/5. Sensation was intact but he was areflexive in both legs. Plantar responses were flexor. Rest of clinical exam was normal. CSF shoed 26WBC/dl (95%), a protein level of 72mg/dl (high), and glucose level was 40% of blood glucose. CSF cx and PCR for numerous viruses were negative. Condition deteroriated in ICU despite treatment with IVIG. By day 4, he ahd lost all lower limb power and was developing sensory abnomrmalities, upper limb weakness, difficulty swallowing and blunting of speech. On day 5, his speech was barely intelligible and his LEFT sided facial nerve palsy of the lower motor neurons (pictured - before and after pt attempts to close eyes and smile). Bilateral upper limb power was significantly reduced to 1/5 but respiratory function remained adequate. He was successfully treated for syphilis in 2005. HIV ab testing was negative at this time but new testing was positive. HIV viral load was >100,000 copies/ml and CD4 count was 408cells/dl. HAART treatment was commenced. Within 12h, his symptoms plateaued and a gradual recovery began. Which of the patient's neuropathies posed the greatest danger?

ANSWER = Acute inflammatory demyelinating polyneuropathy (AIDP) Affecting proximal > distal limbs. True AIDP should have responded to IVIG but he got worse. This plus his history which makes you concerned for HIV1. He did also have mononeuritis multiplex CN VII but not life threatening.

QUESTION (Picture on next side) = A 42-year-old right-handed male police officer presents with a nine-week history of neck, right shoulder and arm pain. He has not noticed changes in gait or balance, bowel or bladder function, or strength. His pain increases when he looks up or rotates right. He has missed several days of work and is unable to sleep at night because of pain. Previous treatment included narcotics and anti-inflammatory medication. On physical examination, he reports pain on palpation of the cervical paraspinals. The neurological exam for muscle strength, muscle stretch reflexes and sensation is normal. Spurling's sign reproduce the patient's symptoms into the arm along the biceps and into the wrist, thumb and index finger. What was the level of the lesion?

ANSWER = C5-C6 Sometimes there are no lateralizing signs of neuropathy in cervical disc disease, only pain. The provocative, Spurling's maneuver suggest that C6 nerve root compression is present (due to the distribution of the radicular pain during the provocative maneuver), so there is nerve root involvement. The Disc above is encroaching on the spinal cord; however, no spinal cord symptoms are present. This case illustrates the fact, that everything does not present in a classical radicular manner. The red flag in radiculopathy is the presence of weakness, people can have increased pain but if no weakness is present it may be managed conservatively (as in this case); if weakness is present then urgent decompression surgery is warranted.

Frank, a 7-year-old boy, often "blanks out" anywhere from a few seconds to 20 seconds at a time. During a seizure, Frank doesn't seem to hear his teacher call his name, he usually blinks repetitively, and his eyes may roll up a bit. During shorter seizures, he just stares. Then he continues on as if nothing happened. Some days Frank has more than 50 of these spells.

Absence (Petite Mal) seizures

You examine a 25 year-old female for migraine headaches. Her examination is entirely normal except you notice a middilated pupil on the right that reacts poorly to light. Some pupillary constriction can be elicited with the accommodation response, but the pupil then remains constricted and dilates very slowly. You also notice that her DTRs in her lower extremities are 0-1/4 bilaterally. Your diagnosis is? 1. Tonic pupil 2. Afferent pupil defect 3. Adie's-Homes syndrome 4. Possible compression of CN III 5. Argyll Robertson pupil

Adie's Homes syndrome = tonic pupil in presence of decreased DTRs of the lower extremities.

A 50-year-old male is seen in your office with the complaint of multiple episodes of visual blindness in his left eye. The episodes are brief, lasting seconds. The patient has no eye pain during the events and feels fine otherwise. He has a past history of angina. Optic neuritis Retrobulbar neuritis amaurosis fugax afferent pupillary defect Papilledema

Amaurosis fugax not optic neuritis - no eye pain not papilledema because vision blindness

A period of several weeks, blurred vision and "droopy eyes" developed in a previously healthy 38-year-old woman. Her symptoms worsened toward the end of each day and often cleared overnight. Friends commented that her speech had become more difficult to understand and softer in volume. She also described increased tiredness on climbing stairs; sometimes she had to stop and rest while cleaning her teeth or brushing her hair. Neurologic examination revealed bilateral ptosis and disconjugate eye movements in several directions of gaze. There was weakness of eye closure and neck flexion and mild proximal limb weakness. What tests confirm your diagnosis?

An edrophonium (Tensilon) test The test resulted in marked improvement for 3 to 4 minutes. Repetitive motor nerve stimulation testing demonstrated a 15% decrease in the ulnar compound muscle action potential amplitude between the first and fifth stimuli. Anti-acetylcholine receptor (AChR) antibodies were present in significantly abnormal titers confirming Myasthenia Gravis. Results of CT scanning of the mediastinum were unremarkable. Immunosuppressive and pyridostigmine bromide (Mestinon) treatments were initiated, with plans for the patient to undergo thymectomy when her symptoms were under control.

A 50 year-old male is seen in your office with the complaint of multiple episodes of visual blindness in his left eye. The episodes are brief lasting seconds. The patient has no eye pain during the events and feels fine otherwise. He has a past history of angina. Based on history alone you suspect; 1. Optic neuritis 2. Retrobulbar neuritis 3. Amaurosis fugax 4. Afferent pupillary defect 5. Papilledema

Answer: (3) Amaurosis fugax. Most likely due to carotid artery disease. Always suspect carotid vascular disease in patients with cardiovascular disease. The key here is sudden vision loss lasting seconds. These people are going to stroke if an aggressive approach is not taken. Remember: Optic atrophy - occurs after optic neuritis and is permanent. Retrobulbar neuritis - the blindness may last for weeks to months or is permanent. Afferent pupillary defect occurs when the patient is blind and no light is transmitted via the optic nerve. Papilledema is a swollen disc and the patients vision is normal or slightly blurred.

A 50-‐year-‐old, right handed male, accompanied by his wife, is seen in your office complaining of numbness and tingling in his feet. You notice he keeps asking you to repeat questions. His wife states that he's becoming more hard of hearing. During the auditory examination you initially notice the patient could hear you rubbing your fingers together better in his left ear. The Weber test lateralized to the left and bone conduction was better than air. The remaining examination was positive for peripheral neuropathy. These auditory findings most likely indicate: 1. A normal patient with peripheral neuropathy 2. A conduction lesion affecting the left ear 3. A sensorineural lesion affecting the right ear 4. Temporal lobe lesion in the right hemisphere 5. Insufficient data to reach a conclusion

Answer: 2 , (go back and review if you missed it)

A patient has weakness of the right face, distal arm > leg. Based on this limited amount of information. Question: Which is likely to be weaker, the forehead or the lower face?

Answer: Lower face. Comment: For a patient to have face, arm, and leg weakness all on the same side of the body, there must be an upper motor neuron lesion which typically spares forehead muscles, because there is bilateral cortical input to the portion of the facial nerve nucleus controlling those muscles. Therefore the lesion is cortical.

"When Bob has a 'drop' seizure, he falls to the ground and often hits his head and bruises his body. Even if I'm right next to him and prepared, I may not catch him. Even with carpet in the bedroom and mats in the bathroom, he gets hurt."

Atonic seizures

A 52-year-old man presents with a 6 month history of progressive weakness. He first noticed difficulty with fine motor coordination of the right hand, but this later progressed to involve the right arm and later all extremities. On examination, he has dysarthria and reduced gag reflex,. There is generalized motor weakness and atrophy. Deep tendon reflexes are hyperactive in both upper and lower extremities. Sustained ankle clonus is present. Fasciculations are seen in the tongue, across the chest, and in all limbs proximally. Sensory examination is normal. There is no family history of neurologic disorders. What is most likely diagnosis? What is the most likely diagnosis in this patient? Progressive muscular atrophy Primary lateral sclerosis Adult-onset spinal muscular atrophy Kennedy's disease (X-linked spinobulbar muscular atrophy) Amyotrophic lateral sclerosis

B, Amyotrophic lateral sclerosis is a progressive, ultimately fatal disorder that disrupts signals to all voluntary muscles. Both upper and lower motor neurons are affected. Other presenting symptoms include muscle fasciculation (twitching), cramping, or stiffness of affected muscles. The earliest symptoms of ALS are typically obvious weakness and/ or muscle atrophy. The parts of the body affected by early symptoms of ALS depend on which motor neurons in the body are damaged first. About 75% of people contracting the disease experience their first symptoms in the arms or legs ("limb onset" ALS). Parents with the leg onset form may experience awkwardness when walking or running or notice that they are tripping or stumbling, ocen with a "dropped foot" which drags gently along the ground. Arm-onset parents may experience difficulty with tasks requiring manual dexterity such as buttoning a shirt, writing, or turning a key in a lock. About 25% of cases are "bulbar onset" ALS. These parents first notice difficulty speaking clearly or swallowing. Speech may become slurred, nasal in character, or quieter. Other symptoms include difficulty swallowing, and loss of tongue mobility. A smaller proportion of parents experience "respiratory onset" ALS where the intercostal muscles that support breathing are affected first. Sensory function generally is spared, as is autonomic and oculomotor activity. ALS is a progressive, fatal, neurodegenerative disease with most affected parents dying of respiratory compromise and pneumonia acer 2 to 3 years; although some die within a year from the onset of symptoms, and occasional individuals have a more indolent course and survive for many years.

A 35-year-old woman with genetically proven myotonic dystrophy presents to your clinic. Her chief complaint is fatigue. She has not seen a physician in several years. She complains of bilateral foot drop and distal hand stiffness but denies any other problems. Social History: She does not smoke or drink. She finished high school at 19 and took one semester of college work. She is recently married. In her family history, she reports that her father wears braces and uses CPAP at night. What do you recommend for the patient?

Baseline EKG (due to cardiac abnormalities), overnight polysomnography, and genetic counseling.

A 38 y.o. homemaker, was brought into the ER by her husband. He reports that she woke this morning, about 10 hours ago, feeling listless, tired. When he returned home for lunch, she complained of blurred vision, a dry mouth, and a lack of appetite. When he came home for dinner, he found her weak, disoriented, and breathing very weakly. Physical exam notes shallow breathing, pupils unresponsive to light, lack of bowel sounds, distended palpable urinary bladder, absent deep tendon reflexes. Mr. Johnson notes that they did nothing out of the ordinary the last several days except go on a family picnic yesterday. They were at a local lake and enjoyed a family carry-in meal consisting of ham, potatoes, fresh and home canned vegetables, and freshly baked banana crème pie. What is your diagnosis? A. myasthenia gravis B. Lambert‐Eaton syndrome C. Lyme disease D. botulism E. cholinesterase inhibitor toxicity

Botulism. Picnic. Weak disoriented. Unresponsive pupils. Botulism Antitoxin. No abx. You admit her to ICU and monitor closely for respiratory failure. You then order Heptavalent (A, B, C, D, E, F, G) Botulinum Antitoxin which is derived from "despeciated" equine IgG antibodies. This is a less immunogenic antitoxin that is effective against all known strains of botulism. The hospital does not keep this in stock and will have to be ordered. You have to keep her alive until it arrives.

A 44yo LEFT handed woman suddenly developed a severe bilateral temporal and occipital HA. She also complained of a stiff neck. When she tried to lie down, she experienced severe N/V twice. She was taken immediately to the hospital where she was noted to be somnolent. She responded appropriately when stimulated and moved all four limbs equally. Her level of consciousness deteriorated over the next 4h to a point where she could not be aroused even with vigorous stimulation. What level? Focal or Diffuse? Mass or Non-mass? What is temporal profile? What general cause?

Brainstem, thalamus, or both cerebral hemispheres; diffuse or multiple levels; Diffuse; Non-mass; acute and progressive; Vascular, Toxic-metabolic, or traumatic.

Needle EMG is contraindicated when: 1. The patient is in Liver failure 2. The patient is in Renal failure 3. The patient's platelet count < 50,000/mm3 4. The patient has Hemophilia 5. The patient is on Heparin or warfarin

C, this is an absolute contraindica8on. The others offer risks but the EMG can be done if the PT and PTT are < 1.5 - 2x normal.

Hemiballismus is an unusual condition characterized by an abrupt onset of violent fleeting movements affecting the limbs, neck and trunk on one side of the body. The most common etiologic diagnosis causing this disorder is: A. Neoplasm B. Trauma C. Vascular D. Degenerative E. Inflammatory

C. Hemiballismus may be viewed as a large-amplitude, violent form of chorea affecting the proximal more than the distal limbs. The most common cause is a lesion of the subthalamic nucleus, most often a hypertensive lacunar stroke.

A 34 year old construction worker noticed six weeks ago, pain in his neck which radiates to his right lateral upper arm. Symptoms have progressively worsened to a point that he is complaining of weakness in his right upper extremity with difficulty carrying heavy objects. Your examination was normal except for weakness in his deltoid and biceps. There was some hyperesthesias noted on his right lateral upper arm. His right biceps reflex was also noted to be 1/4. What was the cause of this complaint?

C4-C5 This is a classic case for C5 nerve root at the C4-C5 disc space. There is deltoid and biceps weakness associated with a diminished biceps tendon reflex.

A 57-year-old man presented with a 3-week history of increasingly severe neck pain with radiation to the right scapula and posterior arm. He had mild numbness with tingling in the index and middle fingers while noting difficulty fully extending his arm. There were no myelopathy symptoms. During the previous 3 years, he had experienced similar intermittent pain and numbness. Neurologic examination demonstrated significant right triceps muscle weakness and an absent right triceps muscle stretch reflex. What is your diagnosis? Upper brachial plexopathy - right Spinal cord lesion - left C5-C6 cervical nerve root- right Cervical meningioma - left C6-C7 cervical nerve root - right

C6-C7 cervical nerve root - right (MRI demonstrated a herniated disc at C6-7. An anterior cervical discectomy and fusion at C6-7 provided immediate relief of his radicular pain. The weakness improved to almost normal within 4 months).

A patient is found to have weakness of abduction of the little finger on the RIGHT hand, reduced pinprick sensation on the palmar surface of the little finger of the RIGHT hand. Where is the lesion?

C8-T1 nerve pathway. Either at peripheral or rostral to where the pathways cross.

In 1960 a young man in his twenties sustained a femur fracture due to an auto accident. Prior to the surgery, he was very adamant about not being "put to sleep" for this surgery because several of his close relatives had previously died under mysterious circumstances while receiving general anesthesia with ether. His anesthesiologist planned to use a just recently introduced new drug, halothane that had the reputation of being safer than ether. Approximately 30 minutes after the patient was asleep and surgery had started, the patient was noted to be very hot to the touch, the urine was found to be of an unexplained brownish color, and blood tests revealed a severely deranged metabolism. The procedure was quickly terminated, the patient was packed in ice for his markedly increased body temperature and then observed in the intensive care unit. Fortunately, the young man survived this incident without any long-term complications. Today we know? 1. You should avoid any general anesthesia during surgery 2. It is an autosomal recessive disorder 3. Affects the ligand-gated sodium channels 4. Can be caused by succinylcholine 5. Treated with Dantrolene a muscle stimulant

Can be caused by succinylcholine Remember malignant hyperthermia is an autosomal dominant defect in sarcoplasmic reticulum (Ligand-gated calcium channel) with massive release of Ca ions producing intense muscle contraction. It is a life threatening condition triggered by exposure to certain drugs during general anesthesia (volatile anesthetic agents or neuromuscular blocking agent succinylcholine). Symptoms include marked hyperthermia > 42C (107F) associated with marked rigidity, rapid heart rate and respirations, hyperkalemia, acidosis, and myoglobinuria. Emergency treatment consists of IV dantrolene (a muscle relaxant), stop anesthesia, and start cooling procedures.

A 50yo woman , an avid gardener, presented with a 2y history of her RIGHT hand "falling asleep." Initially, this occurred only in the morning on awakening. Three months before evaluation, these symptoms began to awaken her at night and were also noted while driving a car or blow drying her hair. She reported that "all" her digits were affected and that her paresthesias were sometimes accompanied by aching of the wrist and forearm. She did not report problems with hand strength. More recently, she was experiencing similar but milder symptomatology in her LEFT hand. Neurologic exam disclosed minimal weakness of RIGHT thumb abduction, without loss of thenar eminence bulk. Results of reflex and sensory examinations were normal, including 2-point discrimination and graded monofilament test. What would you expect to find on the examination of her RIGHT upper extremity? What is the diagnosis? What is the most characteristic clinical feature for the diagnosed condition?

Carpal tunnel syndrome. An early symptom is asymmetric nocturnal hand paresthesia. On neuro exam, her symptoms were reproduced by nerve compression maneuvers , such as nerve percussion (tinel sign) over the median nerve at the wrist.

A 64yo man is referred to a neurologist for a 3y history of progressive weakness in his lower extremities resulting in frequent falls, difficulty standing from a chair, and painful burning in his feet and finger tips. The patient has a past med hx of HBP. His physical examination is significant for weakness in his lower extremities, and weakness of his finger extensors. The sensory examination is dsignficant for loss of sensation in his extremities worse in the toes and fingers and extending above the knee and at the wrist. The DTRs are absent in the upper and lower extremities. The rest of the nruologic exmaination is normal. His lab and MRI studeies of the spine were normal. EMG and NCS confirm a sensory and motor polyneuropathy involving both is lowera nd upper extremities. What is the diagnosis?

Chronic inflammatory demyelinating polyneuropathy (CIDP). This should be considered in pts with progressive symmetrical or asymmetrical polyneuropathy that is relapsing and remitting for ≥2mo. Sensory symptoms, proximal weakness, areflexia without wasting or preferential loss of vibration or joint position are especially suggestive.

A 56yo football coach was evaluated for RIGHT foot drop first noted 6wks earlier when he tripped over his RIGHT foot. Tripping subsequently occurred with increasing frequency, especially when walking on uneven surfaces. He had no recent trauma or illness and no back, buttock, or radicular leg pain. Numbness developed over the lateral lower leg and dorsum of his foot. His medical hx was unremarkable; there was no personal or family hx of diabetes or nerve palsy. Examination revealed a healthy man with normal appearing lower extremities. However, he reported tenderness to palpation at the proximal lateral knee where there was an associated fullness without a discrete mass. Pain was not reproduced with various joint manipulations, all of which were normal. Motor examination disclosed weakness in the RIGHT toe extension, foot dorsiflexion, and eversion. Plantar flexion and inversion of his RIGHT foot, knee flexion, and hip abduction were totally preserved. Sensation was diminished to pinprick and monofilament touch on the dorsum and first web space of the RIGHT foot. He had normal muscle stretch reflexes. What does he have and where is the cause located?

Common peroneal nerve neuropathy w/ compression at the popliteal fossa (due to large cyst)

"Harold's spells begin with a warning; he says he's going to have a seizure and usually sits down. If I ask him how he feels, he just says 'I feel it.' Then he makes a funny face, a mixture of surprise and distress. During the seizure he may look at me when I call his name but he never answers. He just stares and makes odd mouth movements, as if he's tasting something. Sometimes he'll grab the arm of the chair and squeeze it. He may also pull at his shirt as though he's picking lint off of it. After a few minutes, when he's coming out of it, he asks a lot of questions. He never remembers his 'warning' or these questions. The seizures make him tired; if he has two in the same day, he often goes to sleep after the second one."

Complex partial seizure

A 70-year-old patient presented with a gradual onset of bilateral leg weakness. He had experienced a sharp stabbing pain in the mid-dorsal spine that radiated along the lower ribs, occurring while attempting to pick up an object from the ground. The patient had been a manual laborer lifting heavy weights for 30 years but had led a retired sedentary lifestyle for the last two years. He had never received treatment for significant back pain. The patient was unable to move both lower limbs voluntarily. There was significant hypoesthesia below the groin crease. The deep tendon reflexes were exaggerated in both lower extremities. His anal sphincter tone was normal. He was catheterized on admission to decompress an insensate and distended bladder. What is the diagnosis?

Conus Medullaris Syndrome. Signs are almost identical to those of the cauda equina syndrome, except that in conus medullaris syndrome signs are more likely to be bilateral. Sacral segments occasionally show preserved bulbocavernosus reflexes and normal or increased anal sphincter tone. The muscle stretch reflex may be hyperreflexic, (i.e., UMN lesion); Babinski reflex may affect the extensors; and muscle tone might be increased (i.e., spasticity). In this case an intradural disc fragment was excised. Patient did not regain ambulatory status at two-year follow-up.

A 62-year-old woman was in an automobile accident 8 months previously. The left front of the car was struck. The woman was wearing a 3-point seat belt. Subsequently, she noted numbness in all of her left fingers except her thumb. She reports no pain in her left upper extremity. In the recent 3 months, she had experienced progressive loss of fine motor control of her left hand. Thirteen years previously, she had had breast cancer treated by lumpectomy and radiation therapy. Her lymph nodes were normal. Neurologic examination demonstrated weakness of all forearm and intrinsic hand musculature. No muscle stretch reflexes were elicited in the left arm, and sensation was diminished in 2nd through the 5th digits. What is the diagnosis and cause of this patient's problems?

Dejerine - Klumpke syndrome - Radiation therapy A brachial plexopathy due to metastasis in a woman with previous breast cancer suggests the cause may be due to possible radiation induced injury, recurrent tumor, or metastatic disease. Usually, Plexopathies related to radiation for breast cancer are painless and associated with lymphedema occurring years later. The interval from the last dose of radiation to the first symptom of plexus disorder varies widely. The average interval range reported is 7.5 months to 6 years. In the above patient her cancer was 13 years previously (outside the average range). The key point here is that cancer causes a painful brachial plexopathy, radiation plexopathy is painless.

A 59-year-old man from Cape Town, Africa presents with progressive lower extremity weakness associated with numbness and paresthesias in both legs, as well as bladder and bowel incontinence. He has a history of diabetes, hypertension, chronic obstructive pulmonary disease with long-standing intermittent use of steroids, and prostate cancer with metastasis to the lower thoracic spina, treated with radiation 12 months prior. His MRI shows T2 hyperintensity in the thoracic spinal cord, with no evidence of compression or extradural mass. Whati s most likely diagnosis?

Delayed radiation myelopathy. Insidious onset of symptoms occurring 6 months (12-15 months) after radiation. Usually presents with spontaneous uncomfortable sensations in the extremities and may be evoked with neck flexion (Lhermitte's sign) as in the transient variety but persists followed by weakness leading to a transverse myelopathy.

A 40-year-old female is seen by you complaining of a 3 month progressive muscle fatigue with climbing stairs and carrying objects. The patient noted a rash on her cheeks, neck, chest and her hands and swelling around her eyes. Review of systems revealed: sensitivity of her fingers to cold temperatures, difficulty swallowing certain foods, and some shortness of breath on exertion. Neurologic examination reveals proximal muscle weakness in the arms and legs. Reflexes, and sensation are normal. Laboratory studies are normal except for elevated CPK of 770 mg/dl. What is you diagnosis? A. polymyositis B. trichinosis C. dermatomyositis D. inclusion body myositis E. neuroleptic malignant syndrome

Dermatomyositis (DM) (rash, CPK) Characterized by painful proximal muscle weakness (as is polymyositis). DM is distinguished from polymyositis by the associated heliotrope (purplish discoloration of the eyelids) and Gottron patches (papular erythematous scaly lesions over the knuckles). DM may occur in the context of systemic sclerosis or other mixed connective tissue disease, and there is an increased incidence of malignancy in patients with DM.

A 10 m.o. boy has been brought into the ER by his mother. Billy is weak and listless. He barely responds to stimuli and his cry is very soft. His respirations are shallow and are somewhat greater in frequency than normal. In taking history from his mother, she notes that Billy started this listlessness yesterday and it has become worse overnight. Billy does not go to daycare or have much contact with other children. He has been on solid food for about 4 months, eating everything presented to him but has had constipation for the last 2 days. As a treat, last month his mother added small amounts of honey to his diet every so often. Your examination reveals fixed slightly dilated pupils, shallow respirations, decreased swallowing and decreased muscle tone. Otherwise the Neuro Exam is normal. What would your next step be?

Do blood tests and stool stests for botulinum toxin. He should be admitted to ICU. You know that the serum is usually negative for toxin and that initial detection of toxin takes 1 to 4 days (anaerobic cultures take up to 6 days for identification). So call for antitoxin for treatment. Note most hospitals do not have this in the pharmacy and will have to be ordered. You will have to monitor closely and intubate if necessary until the antitoxin arrives.

A 40 year-old-female presents to your office with a chief complaint of a 2 month history of mid thoracic back pain with radiation bilaterally. Over the past 2 weeks she has noticed increasing numbness in the buttocks extending to the perianal region associated with increasing incontinence and weakness in her legs. The exam demonstrated 2+/4 reflexes in the lower extremities associated with 3/5 weakness in muscles of the lower extremities. All other findings on the neurologic exam were normal. Given the history you should suspect: 1. Intradural primary tumor 2. Extradural primary tumor 3. Intramedullary metastatic tumor 4. Intradural metastatic tumor 5. Extradural metastatic tumor

E, Extradural metastatic tumor. In Extradural tumors radicular pain is often prominent at presentation in 90% of patients. As the tumor expands there is early sacral sensory loss associated with spastic weakness. This is due to the superficial location of the leg fibers in the corticospinal tract. The patient in this case was found to have metastatic breast disease. Incontinence and decreased sensitivity in the saddle area (buttocks) are generally considered warning signs of spinal cord compression by the tumor. In a general hospital population, up to 1/3 of patients presenting with CNS metastasis do not have a know underlying cancer. In 30% of patients no primary tumor can be identified, even after extensive evaluation.

A 54 year-old-male presents for an EMG of his right wrist. EMG findings of the right upper extremity includes a median motor distal latency of 5.0 ms. The right sensory distal latency is 4.0 ms. The ulnar motor distal latency is 4.0 ms and the sensory distal latency is 3.0 ms. The nerve conduc8on veloci8es of the median nerve is 49 m/s and the ulnar nerve is 47 m/s. You conclude from this study that: A. The nerve conductions are normal B. The median distal motor latency is prolonged C. The median distal sensory latency is prolonged D. A&B E. A,B&C

E, The NCVs were normal in both the median and ulnar nerves. The only abnormal findings were prolonged latencies of the median motor and sensory nerves. The ulnar latencies are normal. This is a typical finding in Carpal Tunnel Syndrome.

You recheck a 40 year old female that your treating for Myasthenia gravis and her treatment regimen has been stable for the past 2 years. However, recently she has been experiencing increased bouts of fatigue. You decide to administer a short duration (minutes) cholinesterase inhibitor to determine if she is experiencing a cholinergic or a myasthenic crisis. Your choice of treatment is?

Edrophonium (duration of action 10 - 30 min). You can give edrophonium and if she is suffering from a myasthenic crisis she will improve and worsen if she has a Cholinergic crisis. You could have looked at her pupils and determined the same thing. Myasthenic crisis pupils are dilated, cholinergic crisis pupils constricted.

A 21yo male student presents with numbness and tingling over the 4th and 5th digits of his RIGHT hand. States he notices it when he studies and he ahs to move his hand and the symptoms improve. HE has no weakness at this time but you notice slightly diminished pinprick of the entire 5th digit and ulnar half of his 4th digit of his RIGHT hand. Which is the most common location causing his problem?

Elbow Cubital tunnel syndrome w/ compression of the ulnar nerve at the elbow.

A 61-year-old woman is admitted to the hospital with a 4-day history of progressive generalized weakness. Her medical history is unremarkable except for congestive heart failure that is being treated with diuretics. She had been constipated for several days and she then had diarrhea after taking laxatives. Her examination was remarkable for generalized mild weakness, hypoactive reflexes, and flexor plantar responses. Cognition, cranial nerves, and sensation were normal. Her routine laboratory studies are not back yet. What is your diagnosis? A. Guillain‐Barré syndrome B. botulism C. myasthenia gravis D. acute polymyositis E. electrolyte disturbance

Electrolyte disturbance. The patient's treatment with diuretics and her diarrhea after taking laxatives would make you first to consider electrolyte disturbance. A generalized weakness with intact cognition, a lack of sensory disturbances, and flexor plantar responses rules out a disorder of the peripheral nervous system. Useful studies include an electrolyte and serum CPK determination and a sedimentation rate.

A 23-year-old college student is hiking Yosemite National Park when they return home they are complaining of a diffuse discomfort and pain in the left shoulder and weakness of their left arm. The arm dangles at the side with fingers slightly flexed and palm facing backward ("Porter's tip position"). The patient notices paresthesias of the lateral arm, forearm and hand with loss of abduction, internal and external rotation of the shoulder, elbow flexion and radial wrist extension. Rhomboids muscles are normal. The biceps and brachioradialis tendon reflexes are absent. There is no sensory loss of the left upper extremity. What is your diagnosis?

Erb-Duchenne Palsy C5 and C6 are involved leaving the patient with numbness of the lateral arm, forearm and hand with loss of abduction, internal and external rotation of the shoulder, elbow flexion and radial wrist extension due to weakness in the extensor carpi radialis longus and brevis. This must be differentiated from C6 root lesion which does not have shoulder weakness. Note that the lesion is distal to the roots and spares the rhomboid muscles which are supplied by a branch from the C5 root and serratus anterior which is supplied by branches from C5, C6, and C7. Another name is "Rucksack palsy," which can be seen in students with heavy backpacks.

A 14-year-old boy presents for examination because his friends tease him about the way his shoulders "stuck out in the back." Further questioning reveals that for the past year, he has noted weakness in his arms and legs, which worsened to the point that he was forced to drop out of the swim team. His uncle had died at an early age from a muscular disease. Examination revealed tapir lips (pouting), pole neck, symmetric weakness, and atrophy of the muscles of the shoulder girdle, hips, and thighs. A muscle biopsy confirmed the diagnosis. What is your diagnosis? A. facioscapulohumeral dystrophy B. Duchenne's muscular dystrophy C. myotonic dystrophy D. peroneal muscular atrophy E. Becker's muscular dystrophy

Fascioscapulohumeral dystrophy

A 72yo man presented with increasing LEFT groin and upper thigh pain that had begun 1wk previously. Deep venous phlebitis was diagnosed and the pt was treated with anticoagulation. Partial thromboplastin time (PTT) and prothrombin time (PT) were well maintained. 4d later, the patient experienced LEFT flank pain radiating into his LEFT thigh that in a few days become more severe and extended below his knee into the medial leg. He could not raise his LEFT leg off the bed. There was no back pain, radicular pain, or sphincter dysfunction. Neurologic examination demonstrated moderately severe weakness of the LEFT iliopsoas and quadriceps muscles, absent ELFT quadriceps muscle stretch reflex, and diminished sensation to touch and pinprick over the anterior thigh and medial leg below the knee. What is the diagnosis?

Femoral nerve compression (hemorrhage at level of left iliacus and psoas muscle in the pelvis).

A 30-year-old man presents for evaluation of gradually progressive lower extremity spasticity, which has worsened slowly over the past 5 years. On examination he has spasticity of the lower extremities, with no evidence of LMN findings and no other neurologic abnormalities. He says that his father had a similar problem, and his younger brother is starting to have difficulty walking. Work-up has been negative, including an MRI of the brain and cervicothroacic spine that was unremarkable, normal vitamin B12 and copper levels, normal CSF studies for infectious and inflammatory conditions, negative HIV and HTLV-1 serology's and a nonreactive VDRL. What is the diagnosis?

Hereditary spastic paraparesis affects only UMNs (spasticity).

A 34 year-old male computer geek got up from his chair and experienced sudden and severe excruciating low back pain. He has been in good health, except for being hypothyroid. He had no radicular pain but a diminished patella on the right was found. He also had difficulty with his right foot inversion. No sensory findings were present. Tenderness to palpation over the lumbar region associated with severe muscle spasm was noted. The patient was in too much pain to attempt any provocative maneuvers. What do you suspect is the cause of this patient's complaint?

Herniated L3-4. Patella gives clue. Herniation at this level is not common. In this case there was no precipitating cause other than the patient sat at his computer 16 hours a day and lacked exercise. There was no sensory finding you might have used but he had a slight inversion weakness. In cases like this, all you can do if you see this patient in the ER is to do a CT scan (making sure there is nothing structurally wrong, e.g., collapsed vertebrae). You, then must treat his pain with pain medicine and muscle relaxants and have him see his physician for an MRI if symptoms don't resolve.

A 55 year old woman experienced severe low back pain 3 months ago after picking up her grandchild. She complained of severe low back pain that radiated to her left lower extremity all the way to the dorsum of her left foot and was characterized as tingling sensations and numbness. She has been refractory to conservative therapy. On physical examination some hypesthesias were noted over the dorsum of her left foot. There was some weakness (4/5) of the dorsiflexors of her toes as well. Straight leg raising was positive on the left.

Herniated disc at L4-L5. Classic presentation of L5 nerve root (L4-L5) level causing weakness of the dorsiflexors of the lateral 4 toes and sensory changes over the dorsum of the foot. OF NOTE: in treating low back with no lateralizing neurologic finding (sensory or motor) the standard protocol is for conservative treatment for 6 weeks before preforming an MRI, if the symptoms do not resolve. However, we do not live in an ideal world and most people will not put up with severe pain that long. On the other hand, most low back pain does respond to conservative treatment consisting of NSAIDs, muscle relaxants, rest, OMT, conditioning exercises and patient education.

A 47-year-old man had a long history of occasional, severe episodes of low back pain. They usually occurred 1 to 2 times per year after activities such as shoveling snow or playing basketball with his teenagers. Treatment included muscle relaxants, simple analgesics, and instruction to "take it easy." He has never lost work because of these episodes. Three weeks previously, he was awakened with severe left sciatic pain radiating from his lumbar spine to his buttock, down the back of his left leg to his ankle. The pain was improved by lying down. Sitting or bending forward while attempting to put his socks on. He limped and leaned toward the right when he walked. Coughing or straining produced an electric sensation along the sciatic nerve. He had not slept or been able to work since the onset. Physical examination demonstrated a positive straight leg sign on the left, normal muscle strength, an absent left ankle reflex, and altered sensation on the lateral aspect of the foot. Where/what is the patient's problem?

Herniated lumbar disc L5-S1 MRI of the lumbar spine revealed a large extruded disc fragment on the left at L5-S1 with pressure on the S1 nerve root. Because of the severe unremitting pain and MRI findings, surgical decompression was recommended. A microhemilaminectomy was performed, the disc was removed, and the nerve root was decompressed. The patient left the hospital with no sciatic pain the next morning. He returned to work at home in 2 days and to his office pain free in 1 week.

A 7-year-old boy presented with recurrent episodes of generalized weakness, usually after hockey games. Each event resolved within 30 minutes after onset, without associated respiratory distress, dysarthria, or dysphagia. At his worst, the boy could not stand or raise his hands over his head. During the winter months, he noted stiffness in his hands and found it difficult to open his eyes quickly after blinking. His mother recalled having similar episodes but none since late adolescence. He noted that his symptoms were controlled by carbohydrate loading before increased activity. Physical examination results were normal. What is you diagnosis?

Hyperkalemic periodic paralysis These symptoms were episodic not constant therefore not a dystrophy or myotonia congenita. An increased serum potassium level of 6.4 mmol/L, confirms his symptoms. His symptoms responded to carbohydrate loading before exercise. Myotonia is a nonspecific finding of the periodic paralyses and myotonic syndromes.

A 29 year-old Hispanic male with no significant past medical history presented to the emergency room with sudden onset paralysis. The patient had gone to bed at 10 pm with no weakness and awoke at midnight unable to move his upper or lower extremities. The weakness was bilateral and involved both the proximal muscles of the shoulders and hips as well as the distal extremities. He had no respiratory or swallowing difficulty and was able to move his neck and facial muscles. He denied any pain or paresthesia. Prior to this episode, the patient had been healthy and denied any recent diarrhea, chest pain, shortness of breath, or weight change. He did report several episodes of waking from sleep with a "racing heart." He did not take any medications and denied use of alcohol or drugs, or significant changes in diet or activity levels. His mother had been diagnosed with hypothyroidism but his parents and brother had no history of similar episodes and no other significant illnesses. On physical exam, the patient's heart rate was 124 and blood pressure was 193/81. He was mildly obese, but otherwise normal in overall appearance. His skin was cool and dry, and the oral mucosa was moist. No jugular venous distension, goiter or lymphadenopathy were appreciated. Cardiac exam revealed tachycardia with a regular rhythm and no murmurs. Examination of the lungs and abdomen were unremarkable. There were no deformities or edema of the extremities and distal pulses were present and equal bilaterally. Neurologic exam revealed flaccid paralysis of all extremities which involved the proximal and distal muscles and included the hips and shoulders. Sensation was intact but deep tendon reflexes were slightly diminished to 3 out of 4 throughout. Cranial nerve function was grossly intact. Routine chemistry, liver enzymes and complete blood count were normal except for a potassium level of 1.6 (3.5-5 mmol/L). Electrocardiogram revealed sinus tachycardia with Mobitz Type 1 atrioventricular block. What is your diagnosis? What do you advise your patient?

Hypokalemic periodic paralysis b/c pts neuro symtpoms resolved within 2h of IV potassium replacement - f/u to determine source of hypokalemia). Tell pt to take KCl and avoid carbohydrate meals and strenuous exercise. Keep in mind that it is not uncommon to find thyroid disorders in pts w/ periodic paralysis.

A 63 year-old patient presented with a stroke that caused weakness in his left face and upper arm which, for the most part has resolved after 3 days. Currently you find that he is still demonstrating some Dysprosody and has some difficulty recognizing familiar objects. You show him a pen, a key, and a paper clip and the patient holds them in his hands and is still unable to name them. You suspect a visual and tactile agnosia. The requirements for making this diagnosis are all of the following except: 1. The lesion is supratentorial 2. The patient must comprehend the act 3. Have a motor system sufficiently intact to execute the act. 4. Cooperate in attempting it 5. Have normal intelligence

IQ plays no role. The patient must have prior knowledge of the objects presented before the stroke occurred.

A 3rd year KCOM student on rotation presents the case of a patient with a brain tumor who is experiencing some frontal lobe difficulties. The student believes that the patient has an ideomotor apraxia. You confirm this by: 1. Ask the patient to protrude the tongue and move it up, down, right, and left and lick the lips. 2. Ask the patient how to use silverware, thread a needle, strike a match and light a candle. 3. Ask the patient to copy geometric figures (a cross or clock face) or construct them out of match sticks. 4. Watch the patient try to get dressed. 5.Ask the patient to rise and walk.

Ideomotor apraxia is more complicated requiring sequential actions.

A 62yo insulin dependent diabetic female was sitting at home and she fell when getting up to go to the bathroom. He states that her RIGHT knee buckled. She also described a mild RIGHT groin pain which radiated into the thigh. She also noticed some numbness in the medial lwoer leg. She has no other complaints. Your exam was normal except for a mild weakness 4/5 of her RIGHT quadriceps and decreased patellar reflex. What is your diagnosis?

Ischemic damage to the femoral nerve Due to vasa nervorum damage from diabetes.

Sarah, a twenty-one year old student at Iowa State University had spent Thanksgiving day with relatives at her grandparent's farm. During her drive back to campus on Friday morning her vision became blurry, and she was forced to pull over to the side of the road. As she sat in her car, her vision worsened. She opened the car hood in hope of attracting aid and tried to relax. In a short time, a highway patrol officer pulled over and approached Sarah. By this time, Sarah was having trouble swallowing and speaking clearly. The officer helped Sarah to his car and rushed her to the emergency room at a nearby hospital. In the ER, Sarah was able to describe her symptoms to a physician. The physician made note of what Sarah had eaten during the last 24 hours and was especially interested in the fact that Sarah's grandmother canned all of her own vegetables. Why didn't the physician prescribe antibiotics?

It would cause the condition to get worse Treatment with antibiotics would release more toxin in the blood stream and could be potentially fatal.The physician observed that Sarah's breathing was becoming labored. She ordered Sarah's blood sampled, her gastrointestinal tract pumped, and a mechanical respirator prepared for use. Fearing that Sarah suffered from a case of botulism, she asked that Sarah's grandparents be contacted and samples of the Thanksgiving meal retained, if possible, and sent to a local clinic for analysis.

A 64-year-old woman presented with a 1-year history of progressive fatigue, particularly trouble walking up and down stairs and difficulty arising from a chair. Lately she had noted slightly slurred speech, transient horizontal diplopia with ptosis, dry mouth, and tingling in the feet and legs. She had a 80-pack-year history (2 pk/day/40yr) of cigarette smoking. Examination abnormalities included mild weakness of orbicularis oculi, neck flexors, deltoids, triceps, and iliopsoas muscles with difficulty arising from a chair. Her speech was slightly dysarthric. Muscle stretch reflexes were absent but displayed post exercise enhancement. What is your diagnosis? A. myasthenia gravis B. myopathy C. Lambert‐Eaton myasthenic syndrome D. facioscapulohumeral muscular dystrophy E. polymyositis

Lambert-Eaton Myasthenic Syndrome (LEMS) Presynaptic neuromuscular transmission disorder. Serum voltage-gated calcium channel (VGCC) antibodies were increased. A chest radiograph showed a right hilar mass; its biopsy revealed small cell lung cancer (SCLC). The patient received chemotherapy and 3, 4-diaminopyridine for symptomatic treatment of LEMS, resulting in some improvement in strength. She died 16 months later from small cell cancer.

A 45-year-old man reported gradually (2 month) evolving right buttock and posterior thigh pain that eventually caused him to limp. Dysesthesias developed over the outer right foot and were exacerbated by walking and direct pressure on the sciatic nerve. Weakness was limited to the superficial peroneal innervated muscle group. Subsequently, the patient experienced fatigue, abdominal pain, nausea and vomiting, and 3-kg weight loss. A serum uric acid level of 17.2 mg/dL (reference range, 4.0-8.5 mg/dL) was the only abnormality on screening blood studies. EMG testing found problems with both peroneal and tibial nerves as well as the sural nerve. Gluteal and Sciatic innervated muscles were abnormal electrically. Paraspinal muscles were normal. What is your diagnosis?

Lumbosacral plexopathy (right) - due to neoplasm Ultrasonography demonstrated hepatosplenomegaly and retroperitoneal masses. Bone marrow evaluation revealed an undifferentiated lymphoma. CT demonstrated a pelvic malignancy invading the sacral plexus. At autopsy, 3.5 months after the onset of symptoms, the sacral plexus was compressed and infiltrated by Lymphomatous masses at the pelvic brim.

A patient is found to have reduced joint position sense in the LEFT foot, reduced pinprick sensation on the palmar surface of the little finder of the RIGHT hand, weakness of LEFT ankle dorsiflexion, and hyperreflexia at the LEFT ankle. Where is the lesion?

Lesion above the level of S1 (if it was depressed reflex then would imply lesion of reflex arc itself w/in spine)

A previously vigorous 85-year-old man, over the past 5 months, developed intermittent painful paresthesia in his anterior thighs. Typically, walking initially precipitated these; they were distance related and occurred after walking the distance of 1 block. If the man stopped to rest for a minute or so, he could continue walking; gradually, his reserve lessened to the point that he often became symptomatic after only 100 feet. His internist found no evidence of vascular insufficiency. The pain became more incapacitating recently and is even present particularly when the patient was at rest or even when he attempted to sit in a chair. On neurologic examination, the patient had mild diminished strength in his iliopsoas, with hypoactive muscle stretch reflexes at both knees, and the subjective hypesthesias over his anterior thighs. He was claustrophobic, and MRI could not be performed. What is this classic presentation?

Lumbosacral spinal stenosis. A CT myelogram (he is claustrophobic, remember) demonstrated severe lumbosacral spinal stenosis causing pseudoclaudication symptom. A lumbar spinal decompression was performed. The patient had a dramatic improvement in the claudication symptoms and, by 6 months, regained his mobility.

A 17-year-old boy presented with severe pain, stiffness, and "hardening" of his forearm muscles while moving the contents of his friend's house. In the immediate aftermath, he noticed that his urine was "Coca-Cola" colored. Hours later, his physical examination results were normal with the exception of the muscle tenderness to palpation. His serum creatine kinase (CPK) level was increased 50 times normal. His BUN was increased, and myoglobin was in his urine. The patient was admitted to the hospital and treated with vigorous IV hydration. His symptoms resolved within several days. A forearm exercise test demonstrated the expected increase in venous ammonia levels without the expected increase in lactate. What is the diagnosis? McArdle's disease Hypokalemic periodic paralysis Hyperkalemic periodic paralysis Myasthenia gravis Myotonic dystrophy

McArdle Disease This case represents a Myopathy Presenting with Exercise Intolerance. Muscle biopsy revealed subsarcolemmal blebs seen on a periodic acid-Schiff-stained muscle biopsy specimen, which are consistent with glycogen excess. Biochemical analysis demonstrated decreased levels of myophosphorylase.

A 45yo overweight bartender was evaluated for sensory complaints of 4mo duration of her LEFT thigh. She described an aching discomfort extending down the lateral aspect of her thigh, intensified by standing or walking. She also described a burning numbness in the area where her hand would rest if put in her front pants pocket. There was associated cutaneous hypersensitivity with an aversion to having clothes or bed sheets rub against her. She was unaware of any other precipitating events. Examination demonstrated an elliptically shaped area of sensory loss on the distal half of her anterolateral thigh. She had no atrophy, weakness, or reflex loss. What is your diagnosis?

Meralgia paresthetica (due to compression of the lateral cutaneous nerve under the inguinal ligament).

A 75yo woman has reduced appetite and ahs lost weight because of poor nutritional intake. Over the past 4wks she ahs noticed a progressive decline in her gait. She also noticed a mild reduction in her memory. Neurological exam shows decreased joint position sense and vibration sense in her upper and lower extremities and a vibratory level at approximately C5. Laboratory studies disclosed macrocytic anemia. What level? Focal or Diffuse? Mass or Non-mass? What is temporal profile? What general cause?

Multiple levels (supratentorial for memory, spinal for gait and sensory); Diffuse; Non-mass; subacute; Toxic-Metabolic (Vitamin B12 deficiency)

"In the morning, I get these 'jumps.' My arms fly up for a second, and I often spill my coffee or drop what I'm holding. Now and then my mouth may shut for a split second. Sometimes I get a few jumps in a row. Once I've been up for a few hours, the jumps stop."

Myoclonic seizures

A 40-year-old man presents with weakness of all four extremities, facial weakness, and dysarthria of several months' duration. Neurologic examination revealed bilateral ptosis; a "hatchet face," depicting the facial weakness; weakness on flexion and extension of the hands, fingers, feet, and toes; distal atrophy of the muscles of the extremities; slow relaxation of the hand grip; and myotonia of the thenar eminence and tongue. In addition the patient demonstrated premature baldness, cataracts, and testicular atrophy. The diagnosis was confirmed by EMG and muscle biopsy. What is your diagnosis? A. Duchenne's muscular dystrophy B. facioscapulohumeral dystrophy C. myotonic dystrophy D. peroneal muscular atrophy E. Becker's muscular dystrophy

Myotonic dystrophy Weakness, dysarthria,

A 35 year old female with a history of Myasthenia gravis delivered a normal 8 pound son. At home the mother noticed transient feeding difficulties, weak cry, breathing difficulties, and increasing floppiness. The mother states he is having trouble feeding and seems to choke at times. At 8 days of age he was admitted for these complaints. No signs of neonatal lupus was observed. His older brother had autism. On examination the infant you note ptosis, pupils were normal, decreased muscle tone and sucking and shallow respirations. Assuming your diagnosis is correct, you would start treatment with?

Neostigmine Used parenterally due to the possibility of aspiration of the infant. The usual dose of neostigmine methylsulfate is 0.15 mg/kg IM or SQ and lasts 2 to 3 hours. Pyridostigmine can only be given orally and lasts 3 to 6 hours. Edrophonium which is used in the Tensilon test last only 5 to 10 minutes. Atropine does not treat MG but is used to limit the side effects of Neostigmine etc.

A 21yo female college student developed a diffuse body rash, fever, and HA. One day later, she began to complain of neck and back pain, especially with neck flexion. After 2d, she developed reduced level of consciousness as well as continued fever. Neuro exam demonstrated an altered level of consciousness and neck stiffness (positive Kernig's and Brudzinski's) otherwise the exam was normal. Functional/Organic? What level? Focal or Diffuse? Mass or Non-mass? What general cause?

Organic; Spinal, Posterior Fossa, Supratentorial; Diffuse; Non-mass; Infectious (meningococcal rash). Inflammatory(rash and fever)

A 24yo woman was in a MVA. When examined by you in the ER, she had complete loss of sensation from the level of the arms downward She could not move her hands or legs and had no sensation below the axilla. She was incontinent of urine. Functional/Organic? What level? Focal or Diffuse? Mass or Non-mass? What general cause?

Organic; Spinal; Focal Midline (b/c of sensory & motor loss); Mass possible; Trauma

A 55-year-old man develops a feeling of stiffness and clumsiness of his left hand, and when sitting still, he observes a tremor involving the wrist and fingers of his left hand. About 6 months later he also begins to scruff his left foot on the floor occasionally while walking. About 1 year later, the tremor has become constant mostly at rest, and he trips occasionally because of misplacement of the left foot. When you see him you notice that he blinks infrequently. You also find a tremor of the left wrist and fingers, mild rigidity on passive manipulation of fine finger movements of both hands. DX? Essential Tremor Parkinsons Orthostatic Tremor Senile Tremor Crebellar(Holmes) Tremor

Parkinsons age, clumsiness, tremor at rest

"I see this colored ball on my right side. It seems to grow until it fills up my whole view. Everything becomes like a dream and I don't feel real. It is the strangest feeling. Sometimes the seizure stops then, and sometimes it goes all the way and I fall to the floor and have a grand mal (tonic - clonic) seizure."

Partial with Secondarily Generalized Tonic - Clonic Seizures

"The seizures start with a tingling in my right thumb. In seconds, my thumb starts jerking. Soon, my whole right hand is jerking. I have learned that by rubbing and scratching my forearm I can sometimes stop the seizure. Other times the jerking spreads up my arm. When it reaches my shoulder, I pass out and people tell me that my whole body starts to jerk."

Partial with Secondarily Generalized Tonic - Clonic Seizures

A 23yo male presents with numbness and tingling of his RIGHT hand. The patient's hand numbness wakes him up at night and he has to shake it for normal feeling to return. There is a positive tinel's sign over the median nerve of the wrist. The patient has no other complaints. What level & what general cause?

Peripheral (Carpal Tunnel Syndrome); Inflammation/Trauma (repetitive) 1. (D) Peripheral. This patient has Carpal Tunnel Syndrome, compression of the median nerve under the flexor retinaculum of the wrist. In mild cases there may be no other physical findings, however, in severe cases there may be atrophy of the thenar eminence may be observed. 2. (E) Based on this history you would expect inflammation (swelling) as the primary cause. If you answered trauma you would also be correct (as most Carpal tunnel syndromes occur as a result of repeated trauma to the wrist as in repetitive use of the hand would be the underlying cause).

A 48yo woman experienced the abrupt onset of pain, followed by paresthesia and loss of feeling n a rather circumscribed area along the lateral aspect of her RIGHT thigh. Neuro exam showed a localized area of decreased perception of pinprick, temperature, and touch. The results of the rest of the exam were normal. What level? Focal or Diffuse? Mass or Non-mass? What is temporal profile? What general cause?

Peripheral (lateral femoral cutaneous nerve); Focal Right; Non-mass; Acute; Traumatic/vascular - Meralgia paresthetica

A 58yo female with a hx of a light stroke 9y ago that left her with short term memory problems. She smoked ½ pack/day for 25y and had intermittent low back pain for 20y. She presents to you with a hx of waking up 3wks ago with a weakness in her RIGHT lower extremity associated with foot drop. It has not progressed or improved. She has dorsiflexion weakness and a numbness form the lateral mid-calf to the dorsum of the affected foot. She ahs no reflex in her RIGHT patella. What level? Focal or Diffuse? Mass or Non-mass? What general cause?

Peripheral; Focal Right; Non-mass; Trauma (compression of the common peroneal nerve over the fibular head causing vascular ischemia). At first glance, the cause is considered to be due to compression (trauma) of the common peroneal nerve over the fibular head causing ischemia (vascular). Additional history finds that the patient is unable to lay on her back and favors sleeping on her right side, with a pillow between her legs, for the past 20 years. Another way to look at it is that she was normal when she went to bed the night before and woke up with a foot drop. Something happened during the night. With her past history of vascular disease ...the cause is vascular until proven otherwise. Additional history might find a history of peripheral vascular disease. On examination you would want to make sure that her arterial pulses were normal in her lower extremities. I HOPE YOU CAN SEE HOW THE HISTORY GUIDES YOUR THINKING AND GIVES YOU AN INITIAL DIFFERENTIAL DIAGNOSIS WHICH GUIDES YOUR PHYSICAL EXAMINATION.

A 46yo assembly line worker noted numbness and pain in the first 3 digits over his RIGHT hand with repetitive use and progressive over the last 3mo. He also had weakness of his RIGHT thumb (opponens pollicis) but not of other muscles of the hand. The history and exam was otherwise normal. What level? Focal or Diffuse? Mass or Non-mass? What is temporal profile? What general cause?

Peripheral; Focal; Non-mass; Chronic; Traumatic (compression - carpal tunnel).

An 18-year-old female presents with a concern that her right pupil is larger than the left. She notices that it also varies from day to day and even from hour to hour and often will disappear. On examination you notice that the left pupil demonstrates the same degree of asymmetry in size in low, ambient, and bright light conditions. DX? Marcus Gunn pupil Adies tonic pupil Physiologic anisocoria Early Horner's Possible compression of CNIII

Physiologic anisocoria varies, if nerve - would be constant Marcus gunn- blind in that eye adies dilated and same Horner's myosis anhydrosis and ptosis

A 20 year-old female presents with a concern that her right pupil is larger than the left. She notices that It also varies from day to day and even from hour to hour and often will disappear. On examination you notice that the left pupil demonstrates the same degree of asymmetry in size in low, ambient, and bright light conditions. Your diagnosis is: 1. Marcus Gunn pupil 2. Adie's tonic pupil 3. Physiologic anisocoria 4. Early Horner's syndrome 5. Possible compression of CN III

Physiologic anisocoria (Benign essential anisocoria). The pupil remains the same in low, ambient and bright light. Reassurance is all that is needed.

A 33yo female was seen due to a 3-4mo gradual onset of symmetrical proximal muscle weakness. No complaints of muscle pain only weakness. She feels fatigued, has morning stiffness and a 10lb wt loss. The exam is otehrwise normal except the proximal muscle weakness. There is no family history of muscle disease. What disease do you suspect? What is initial testing?

Polymyositis. Do ESR, CRP, CPK, aldolase, thyroid panel (want to r/o thyroid disease and rule in Polymyositis). Polymyositis is often associated with other autoimmune diseases, including Crohn's disease, vasculitis, sarcoidosis, and Myasthenia gravis.

A 65yo male with a hx of HTN and CAD presents wit ha complaint of sudden onset of double vision. Further questioning reveals the diplopia occurs only on horizontal gaze and not vertical gaze. Other than weakness of the RIGHT lateral rectus muscle, there are no other neurological complaints or findings. What level & what general cause?

Posterior fossa (ddx from peripheral nerve b/c has additional long tract findings w/in rectus); Vascular (lesion at 6th cranial nerve w/in brainstem) 1. (B) and or (D) The patient is suffering from a peripheral nerve lesion involving the 6th cranial nerve located in the posterior fossa. Remember in the posterior fossa we try to distinguish between a lesion of the peripheral cranial nerves (after exiting the dura) and those within the brainstem (with additional brainstem or long tract findings). In this case there were no other brainstem findings indicating that only the right lateral rectus muscle was weak; thus a peripheral lesion. I would accept either posterior fossa or peripheral level as an answer as long as you know the distinction that I have illustrated. 2. (A) Vascular due to the sudden onset. Also his history of hypertension and coronary artery disease is highly suggestive that the patient has multiple vascular problems.

Over several years, 50yo male noted the onset of ringing in his RIGHT ear and then loss of hearing in that ear. He also experienced RIGHT facial weakness and decreased sensation. In the weeks before you examined him, he noted stiffness and weakness of his LEFT arm and leg. What level? Focal or Diffuse? Mass or Non-mass? What is temporal profile? What general cause?

Posterior fossa (due to V, VII, and VIII and opposite long tract signs); Focal; Mass; Chronic; Neoplasm - Acoustic neuroma.

A patient is found to have reduced pinprick sensation on the LEFT upper and lower face, reduced pinprick sensation on the palmar surface of the little finger of the RIGHT hand? Where is the lesion?

Posterior fossa (ipselateral face, contralateral body) Intersection points of these pathways lie in the region extending from the left mid-pons down to the left side of the spinal cord at the C2 level.

"I almost enjoy them. The feeling of déja vu, as if I've lived through this moment and I even know what's going to be said next. Everything seems brighter and more alive."

Simple partial seizure

A 17-year old male, is quarterback of his high school football team. Recently he has experienced dramatic fatigue after only brief episodes of exertion. Your tests indicate Billy is suffering from myasthenia gravis. You decide to use a cholinesterase inhibitor to improve Billy's condition. You want to use a cholinesterase inhibitor with a duration of action of about 3-6 hours, and one that will not easily distribute to the CNS, to decrease the possibility of adverse effects. The best treatment is?

Pyridostigmine (Mestinon) Universally used as the treatment for generalized Myasthenia gravis. The majority of patients can be maintained on this for ever. Some patients seem to build up more antibodies and the pyridostigmine does not seem to work as effectively, thus; the patient will have to start on immunosuppression with either prednisone or azathioprine for maintenance in addition to the pyridostigmine.

ANSWER: CN III palsy. She should be referred to an emergency department immediately for a neurosurgery consultation for possible intracranial aneurysm, magnetic resonance imaging and angiography of the brain, and possibly a cerebral angiogram. The third cranial nerve travels within the subarachnoid space between the posterior cerebral and superior cerebellar arteries. This is also near the posterior communicating artery, an area prone to the development of vascular brain aneurysms. This patient has paresis of five extraocular muscles, which is the cause of the droopy eyelid and paralysis of all eye motility except abduction (innervated by CN VI). The pupil is dilated because the parasympathetic fibers that cause pupil constriction travel on the outside sheath of the third cranial nerve and are highly susceptible to compression by an adjacent aneurysm.

QUESTION = A 37-year-old woman presents to an urgent care clinic with acute onset of a headache and a droopy left eyelid. Visual acuity is 20/20 in each eye. When you lift up her left upper eyelid, you discover that the eye is deviated outward and downward (see image above). When you assess ocular motility, the left eye can't elevate, adduct or depress. The only motility present in the eye is the ability to look away from the nose. The left pupil also appears dilated compared to the right pupil. The right eye shows no motility deficits. What is your diagnosis? 1. Internuclear ophthalmoplegia 2. Optic neuritis 3. Third-nerve palsy 4. Fourth-nerve palsy 5. Myasthenia gravis

ANSWER = The physician sees a swollen disc and the patient sees nothing (blind); due to optic neuritis (papillitis).

QUESTION = Does the patient complain of visual loss? What is this?

ANSWER = The physician sees a swollen disc, and the patient sees as usual; due to papilledema

QUESTION = Does the patient complain of visual loss? What is this?

ANSWER = The patient usually has a reduction in visual acuity. This is optic atrophy which is a result of a healed optic neuritis commonly seen in multiple sclerosis. The atrophy occurs after the neuritis heals and remains for life.

QUESTION = Does the patient have normal visual acuity? What is it and what is the cause?

ANSWER = The physician sees nothing (a normal disc) and the patients sees nothing (blind); due to acute Retrobulbar neuritis. If the patients vision is normal this fundus is normal (this was a trick question, I just couldn't help myself).

QUESTION = If the patient cannot see, what is the cause? If the patient can see what does this represent?

ANSWER = The patient has a trochlear nerve palsy affecting the right eye. Remember: the head tilt is opposite the affected eye.

QUESTION = The above patient presents with blurry vision or some vague problem when looking down, especially when reading a book or climbing stairs. Which eye is affected?

ANSWER = (A) Positive Scotoma - blackness or a sense of blockage of vision which suggests retinal diseases. (B) Negative Scotoma - absence of vision a blank spot, suggests optic nerve lesions.

QUESTION = The above pictures A and B represent what? And what is their significance?

ANSWER = A. The patients left eye B. CN III C. Microvascular injury to the nerve (pupils are hard to see but are normal). D. Patch the good eye and symptoms usually improve within 6 weeks. Answer: Diabetes and hypertension

QUESTION = This 60 year-old man presents with the diplopia and the above abnormal eye movements. It occurred suddenly and although the picture doesn't clearly demonstrate the pupils are equal and reactive. He has no previous neurological history of any abnormality. A. Which eye is abnormal B. Which nerve is abnormal C. What is your diagnosisp D. What is your advice to the patient? What is the most common cause of this condition?

"It is a pressure that starts in my stomach, then rises to my chest and throat. When it reaches my chest, I smell an unpleasant odor of something burnt. At the same time I feel anxious."

Simple partial seizure.

ANSWER = Answer: Isolated trochlear nerve palsy (in the left eye) no cause was found, he has probably congenital and later decompensated. Answer: to his right. Head tilt will be away from the affected eye; his left eye is abnormal

QUESTION = This patient presents with the following history: During college when I'd be tired or staring off into space I'd get brief double vision, which I could correct by just shifting my head around a bit until my eyes corrected it. Otherwise, I seemed fine. After college and getting a job waiting tables, things started to get different. Coworkers would comment on me tilting my head and staring at things, something I didn't even realize I was doing. Then I got a job working for an insurance company. The training and the fear of losing this job were stressful to me. About a week or two in, after spending hours reading manuals, looking at old CRT terminals, and staring at blackboards, I began to see double. In the past, a bit of head shaking and tilting would have corrected it. Now, nothing, short of covering one eye with paper would allow me to see one image. It freaked me out. I'm not normally one with health anxiety, but I started to think, "brain tumor!" I could no longer drive (it's tough to drive when two lanes of traffic looks like four). What is your diagnosis? Which way did the patient tilt his head to decrease his diplopia?

ANSWER = Occipital lobe lesions that spare the posterolateral striate cortex causes a contralateral homonymous hemianopias that are symmetrical, and tend to spare the macula.

QUESTION = You suspect a lesion in the 1. Occipital lobe 2. Optic radiations 3. Optic tract 4. Optic chiasm 5. Retina

A 4 year-old young male patient presents to your office with his mother who states that he has trouble going up and down stairs. Mother states that he was able to walk without support at 1 year, although he began to lose this ability. He has some pseudohypertrophy resembling Duchenne's muscular dystrophy but this has been ruled out by previous tests. Given this brief history you must include which of the following in your differential diagnosis: A. SMA type 0 B. SMA type 1 C. SMA type 2 D. SMA type 3 E. SMA type 4

SMA type 4 = there are several diseases that present in children with muscle weakness including the infamous Duchenne's Muscular dystrophy. Spinal muscular atrophy type 2 (Kugelberg-Welander disease) due to anterior horn cell disease, has to be considered in the differential in this age group. (Remember the age of onset may be the most important fact in this diagnosis).

Following a bout of alcoholic intoxication, a 25yo man developed weakness of the RIGHT wrist and finger extension and RIGHT brachioradialis, with preservation of the triceps and deltoid function. What is the most likely diagnosis? Another condition that may present with similar symptoms is?

Saturday night palsy. Lead intoxication (lead loves the radial nerve)

A 68yo man presented with a 3y history of trembling of the legs. The rumor began immediately after he stood and subsided when he began walking. His medical history was irrelevant to his problem. Neurologic examination showed fine, rapid tremor of the legs that began immediately after the patient rose to the standing position and that abated on walking. Sensation of pain and head was impaired in a stocking distribution, and the achilles tendon reflexes were absent.

Shaky leg syndrome (aka orthostatic tremor)

A 22-year-old female college soccer player has been treated for intermittent occipital headaches for the past year. The headaches were noted especially when looking up. Occasionally when sneezing, she will experience a sharp, electric pain radiating down her neck to her extremities (Lhermitte's sign). Over the past 3 months she has noticed a decrease in temperature sensation (when taking a shower) over her shoulders and arms (in a cape distribution). The examination confirms a decrease in temperature and pain in the areas described associated with areflexia in both upper extremities. All other neurologic findings were normal. What is the diagnosis?

Small central cord syndrome (syringomyelia) associated with Chiari type 1 malformation of the cerebellum. The patients headaches have been noticed when hyperextending the neck causing the cerebellar tonsils to compress against the spinal cord. Any sudden valsalva maneuver can produce Lhermitte's sign. The development of the loss of pain and temperature in a cape distribution of the shoulders and arms is pathognomonic of Syringomyelia. This problem usually requires surgery to enlarge the foramen magnum to allow the space necessary for the cerebellum to avoid compressing the spinal cord.

A 63yo male with a five year history of recurrent low back pain and bilateral posterior thigh and calf pain which devleoepd after a lifting injury. At the tiem of onset he went ot see a chiropractor and manipulations minimally improved his low back pain. ON presentation he complained of sharp lower lumbar pain with occasional bilateral posterior thich and calf pain. He describe also a sharp burnign pain and numbness of the toes and plantar aspects of both feet. Lifting abd bending, as well as prolonged sitting, standing, or lyin in one position aggravatined his pain and numbess. He was not able to walk more than tree city blocks without aggravating his back pain or producing numbness in the feet. The patients was in fairly gfood general health, but had been a borderline diabetec taking medication for the last year and a half. On examination the patient was bent over and extension of his lumbar spine caused pain. There was tenderness at the L4-5 and L5-S1 posterior joints, especially on the leg. Straight leg raising was to 90 degrees bilaterally. There were no nerve root tension signs on provacative maneuvers. Muscle strength was graded 5/5 in the lower extremi6es. The patient had a subjective sensory decrease to light touch within the L4 distribution on the right side. The toes, dorsum of foot and plantar aspect of the forefoot were numb bilaterally, especially on the right foot. No deep tendon reflexes could be elicited in his lower extremities. The plantar responses were flexor. Vascular examination of the lower limb revealed clinically diminished dorsalis pedis and posterior 6bial pulses bilaterally. Both feet showed trophic -changes at the toes, that included dryness and scaliness of the skin, hair loss and nail changes. Rubor was evident bilaterally in the dependent posion. You suspect:

Spinal Stenosis. This case report points out the importance of a thorough history, and a comprehensive examination during the evaluation process to rule out vascular claudication that might mimic neurogenic claudication. There three strict criteria for the diagnosis of vascular interment claudication: 1) The pain must come on as a muscle cramp in thigh or calf after the pt has walked a predictable distance. 2) It must be relieved by rest in the standing position after a predictable period of time. 3) The pt must then be able to walk a similar distance again, and again obtain relief after resting the same period of 6me as before.

A 19-year-old college football player experienced a severe collision with another player, resulting in a hyperextension injury of his cervical spine. He noticed bilateral transient burning sensation in his upper extremities, especially his hands. Symptoms lasted 30 minutes but soon resolved. What is the diagnosis? Incomplete posterior cord syndrome Cervical plexus lesion Spinal cord concussion C5-C6 nerve root lesion A "stinger"

Spinal cord concussion. This is better known as "Burning Hand Syndrome" which occurs as a result of a hyperextension injury resulting in a spinal cord concussion. Symptoms of paresthesias and dysesthesias (burning pain) are bilateral, transient and may last from seconds to hours. This implies that the spinal cord not the brachial plexus (unilateral) is involved.

A 21yo soldier returned from Iraq after sustaining a GSW in his spinal column. On neurologic examination, you note that he has weakness of the lower LEFT extremity. IN addition, he has lost a pin and temperature perception on the RIGHT side from about the level of his navel downward. All else normal. What level? Focal or Diffuse? Mass or Non-mass? What is temporal profile? What general cause?

Spinal; Focal left; Non-mass; Acute; Trauma - Brown Sequard Syndrome at the T10 level (remember the spinothalamic tracts cross 2 levels above).

A 55yo woman has been brought to the ER by her husband because she seems confused and his having progressively more difficulty expressing her thoughts. For at least the last 10wks she has had increasing clumsiness and weakness in her RIGHT arm and RIGHT leg, and she is bumping into objects in her home. On examination, she has a RIGHT homonymous hemianopia, aphasia, a mild RIGHT hemiparesis (face, arm, leg), and increased reflexes on the RIGHT side with a (+)Babinski's sign on the RIGHT. What level? Focal or Diffuse? Mass or Non-mass? What is temporal profile? What general cause?

Supratentorial (Left cerebral cortex); Focal Left; Mass (progressive nature); Chronic; Neoplasm

A 69yo RIGHT handed retired executive is seeing her internist for a routine checkup. Her husband mentions that she has undergone a marked personality change over the past several months. He also notes that she has been forgetful for about a year and keeps asking the same things over and over. She no longer seems interested in her personal appearance. The mental status exam confirms these observations. Her speech is fluent, but she frequently pauses because she cant think of a word. She has difficulty following complicated commands. She can only remember one of three items after a 5minute delay, and recalls no current events. She is unable to subtract two-digit numbers in her head. The remainder of her exam is normal. What level? Focal or Diffuse? Mass or Non-mass? What is temporal profile? What general cause?

Supratentorial (cortical); diffuse; non-mass; chronic; Degenerative or toxic-metabolic.

A 23yo graduate student was studying late at night for an exam. As he looked at his textbook, he realized he was having difficulty reading through his LEFT eye. When he covered his LEFT eye, his vision in the RIGHT eye seemed normal. However, when he covered his RIGHT eye, his vision in the LEFT eye was blurred. Within several hours he lost his vision completely. Other than LEFT ocular pain when he moved his eyes, he had no other symptoms. What level & what general cause?

Supratentorial (optic nerve); Inflammatory (optic neuritis, multiple sclerosis) 1. (A) Supratentorial. The visual pathways including the optic nerve, tract, radiations and occipital lobe are above the tentorium. This case associated with eye pain on movement and visual loss points to a lesion of the optic nerve. 2. (E) If you guessed inflammatory you would be correct. This case is classic of Optic neuritis and given the age of this patient, you will learn later on, you would suspect Multiple Sclerosis.

A 2yo child was seen by you at 1pm due to stumbling and lethargy associated with N/V. The mother relates the child was in good health this a.m. but was difficult to arouse from his nap for lunch. You find the patient to be somnolent, arouses to light pressure but falls back to sleep and his pupils are constricted but reactive to light. The temp is normal and no other neurological signs are present. What level? Focal or Diffuse? What general cause?

Supratentorial and/or posterior fossa (level of consciousness); Diffuse (no localized signs); Toxic/Metabolic 1. The answer is (A) and or (B) Supratentorial and posterior fossa are affected due to the level of consciousness. Remember the reticular activating system? Certainly diffuse processes affect all of the nervous system however the most alarming problem is the level of consciousness. You will have to examine the other parts of the nervous system when the patient wakes up. 2. (D) Diffuse, as you cannot find any localizing signs. 3. Toxic/metabolic is most likely. At this age the patient probably has taken some form of toxin or medication and your workup should be toward uncovering the type of substance followed by appropriate treatment. The pupillary dilation/constriction will help guide you as to the class of substance taken.

A 19yo male was in a MVA. 2wks later, he gradually developed progressive HA and personality changes, with reduced motivation. His family also noted that his RIGHT face seemed to droop and he had mild weakness in his RIGHT arm and leg. What level? Focal or Diffuse? Mass or Non-mass? What general cause?

Supratentorial; Focal Left; Mass (hematoma at least until proven otherwise); Subdural hematoma s/p MVA (take up to 3wks to form) Given the history of trauma he most likely, until proven otherwise, has a subdural hematoma due to the auto accident. (Subdural hematomas may occur up to 3 weeks after the trauma)

A 60yo male presents with a 3mo gradual decline of strength in his RIGHT face and RIGHT upper and lower extremities. He first noticed a weakness in his face and then his hand that progressed for 2mo. This was followed by a progressive weakness in high RIGH leg to a point where he is now falling. What level? Focal or Diffuse? What general cause?

Supratentorial; Focal Left; Neoplasm (remember anything focal, chronic, and progressive - think cancer!)

A 28yo accountant and part time boxer has been brought to the urgent care by his wife because he has become more irritable and abusive. She reports that he has had intermittent HA for 3mo, and the HA have become more severe and constant in the past month. He has been unable to work for about a week because of excessive drowsiness, and he sleeps for up to 24h if not awakened. The patient is poorly cooperative and says only that he has a HA. On neuro exam, he is drowsy and irritable but able to follow commands. He has mild to moderate weakness in his LEFT arm and leg and a LEFT pronator drift. Tendon reflexes are hyperactive on the LEFT with a Babinski response. The lower part of his face droops slightly on the LEFT side. What level? Focal or Diffuse? Mass or Non-mass? What is temporal profile? What general cause?

Supratentorial; Focal Right; Mass; Chronic progressive; Neoplasm

60yo male comes to see you for numbness and a slight weakness in his LEFT lower face. He states it was present on awakening yesterday morning and it was not present when he went to bed. On further questioning he admits that he has noticed numbness and clumsiness in his LEFT hand when buttoning his shirt. 1. Which level do you suspect his problem is located? A. Supratentorial B. Posterior fossa C. Spinal D. Peripheral 1. Based on this history alone, what do you think the cause is? A. Vascular B. Neoplasm C. DegeneraKve D. Toxic\metabolic E. Inflammatory F. TraumaKc

Supratentorial; Vascular 1. (A) Supratentorial because the numbness is in the right face and hand (contralateral to the focal lesion). Those of you who answered posterior fossa may have been confused by the term clumsiness, thinking it was cerebellar in origin. NOTE: People may feel clumsy in a weak extremity (have you ever had your arm or leg fall asleep? What happened to you until the feeling and strength came back? You probably felt clumsy but your cerebellum was OK). You must ask further questions to determine if the patient's statement 'my right hand is clumsy' they mean weakness or incoordination. 2. (A) If you guessed 'Vascular' your correct. As you will learn later in this course the patient most likely had a thrombotic event during the night. You can also infer that his problem was in the distribution of the middle cerebral artery.

A 51yo male presented with a 5y history of intermittent numbness in his LEFT foot on the medial aspect. Recently pain has developed and he stated it's a "sharp knife-like pain" associated with numbness on the sole of his foot. The pain wakes him at night on occasion. He describes the pain increased the more he was on it, mostly in the afternoon and that it was not as bad at the beginning of the day. The neurological exam was normal except pain was elicited with inversion of the foot. What is the diagnosis?

Tarsal tunnel syndrome Pain is worse as day progresses, pt may or may not have positive tinels sign over the nerve as it passes under the flexor retinaculum.

A patient is found to have reduced joint position sense in the LEFT and RIGHT foot and LEFT and RIGHT hand. Where is the lesion?

The findings are bilateral; therefore a unilateral lesion could not involve all four pathways, a bilateral lesion centered on the midline and extending out to each side could explain the pattern of deficits. The sites where this is possible are in the lower medulla, cervical cord (at the level of C7 and above) and the post central gyrus of the cortex.

A patient is found to have reduced joint position sense in the LEFT and RIGHT foot and LEFT and RIGHT hand, normal strength and sensation proximally in all four limbs. Where is the lesion?

The first four symptoms imply a lesion site lying in the midline and extending bilaterally either in the cortex or in the region extending from the low medulla to the C7 level of the spinal cord. No single lesion site can explain all of the findings in this patient. This patient has a multifocal or diffuse condition. A characteristic unifying all of the lesion sites is sensory deficits restricted to the distal portions of the limbs. This suggests a process affecting peripheral nerves, specifically involving the longest sensory nerves. Peripheral polyneuropathy is the term for such a condition. Today in the United States the two most common causes for this condition is Diabetes and Alcoholism.

A patient if found to have reduced joint position in LEFT foot, reduced pinprick sensation on the palmar surface of the little finger of the RIGHT hand, reduced visual acuity in the LEFT eye. Where is the lesion?

The first two items are the same as in the previous question and imply a lesion localized somewhere between the low medulla and the C6-C7 level of the spinal cord on the left. This region is not even close to the visual pathway representing the patient's visual loss. This means that no single lesion site could produce all of this patients findings. By the rules of the game, the next objective is to explain all of the deficits on the basis of two lesion sites. Therefore one lesion is in the region of the medulla or spinal cord and the other lesion is in the left retina or optic nerve. The next step is to decide if the two lesions are related (i.e. in MS).

"These seizures frighten me. They only last a minute or two but it seems like an eternity. I can often tell Heather's going to have one because she acts cranky and out of sorts. It begins with an unnatural shriek. Then she falls, and every muscle seems to be activated. Her teeth clench. She's pale, and later she turns slightly bluish. Shortly after she falls, her arms and upper body start to jerk, while her legs remain more or less stiff. This is the longest part of the seizure. Finally it stops and she falls into a deep sleep."

Tonic-Clonic (Grand Mal) Seizure

A 19-year-old man was involved in an automobile accident 4 months earlier. He sustained only minor bruises about the head and face. When he returned to school in the fall, he seemed to be uninterested in his schoolwork and began to complain of headaches. He dragged his left foot when he walked and used his left hand clumsily. He had a slight droop to the left side of his face. Level? Process? Cause?

supratentorial, focal right, hematoma?

A right handed 68-year-old male awakened one morning and noted that he was unable to speak clearly. He wanted to ask for help but could utter only the words "Ago now". His wife noted some weakness of the right side of his face and right arm and leg. He seemed unable to answer the questions his wife posed to him. Level and location of lesion? Cause?

supratentorial, focal‐left, stroke


Conjuntos de estudio relacionados

The Emotion Code Definitions of Emotions

View Set

Chapter 5, Alternatives to Experimentation: Correlational and Quasi- Experimental Designs

View Set